The superconducting circuit companion – an introduction with worked examples

S. E. Rasmussen,1, ∗ K. S. Christensen,1 S. P. Pedersen,1 L. B. Kristensen,1 T. Bækkegaard,1 N. J. S. Loft,1 and N. T. Zinner1, 2, † 1Department of Physics and Astronomy, Aarhus University, DK-8000 Aarhus C, Denmark 2Aarhus Institute of Advanced Studies, Aarhus University, DK-8000 Aarhus C, Denmark (Dated: March 3, 2021) This article is a tutorial on the quantum treatment of superconducting electrical circuits. It is intended for new researchers with limited or no experience with the field, but should be accessible to anyone with a bachelor’s degree in physics or similar. The tutorial has three parts. The first part introduces the basic methods used in quantum circuit analysis, starting from a circuit diagram and ending with a quantized Hamiltonian truncated to the lowest levels. The second part introduces more advanced methods supplementing the methods presented in the first part. The third part is a collection of worked examples of superconducting circuits. Besides the examples in the third part, the two first parts also includes examples in parallel with the introduction of the methods.

CONTENTS 4. Lagrangian and Hamiltonian 24 C. Dissipation 24 Introduction 1 1. Dissipative elements 24 2. Adding impedances to the equations of I. Basics 2 motion 25 A. Analyzing lumped-element circuit diagrams 2 3. Single impedance 26 1. Circuit variables 3 4. The Caldeira-Leggett model 26 2. Circuit components 3 5. Master equation 27 B. Method of nodes 5 6. Two level approximation 29 1. Equations of motion 6 2. Obtaining the Hamiltonian 7 III. Examples 30 3. Normal modes 8 A. Phase qubits 30 4. Change of basis 8 B. Charge qubits 31 C. Quantization and effective energies 9 1. Single Cooper pair box 31 1. Operators and commutators 9 2. Transmon qubit 32 2. Effective energies 9 C. Flux qubits 33 D. Recasting to interacting harmonic oscillators 10 1. DC-SQUID 33 E. Time averaged dynamics 11 2. C-shunted flux qubit 34 1. Interaction picture 11 3. Fluxonium 34 2. Rotating wave approximation 11 4. 0-π qubit 35 F. Truncation 13 D. Coupled qubits 36 1. Two-level model 13 1. Gmon coupler 36 2. Three-level model 14 2. Delft coupler 37 3. Exact truncation 15 E. Coupled qutrits 39 4. Exact four-level model 16 1. External driving of the modes 40 G. Driving the qubit 17 F. Multi-body interactions 41 1. Generalization to qudit driving 18 1. External coupling to eigenmodes 42 H. Overview of the basic method 19 arXiv:2103.01225v1 [quant-ph] 1 Mar 2021 IV. Summary and outlook 42 II. Advanced 19 A. Fundamental graph theory of electrical Acknowledgments 43 networks 19 1. Circuit matrices 21 References 44 B. Method of electrical network graph theory 22 1. Kirchhoff’s laws 22 2. Equations of motion (without dissipation) 23 INTRODUCTION 3. Voltage and current sources 24 Since Richard Feynman first proposed to use quantum simulators to simulate physics [1, 2], an increasing amount of attention has been given to quantum processor and ∗ [email protected] quantum technology, something which is only expected to † [email protected] increase further in the coming years [3–5]. This increased 2 attention has led to swift progress within the field of In Section II we present more advanced methods. These quantum mechanics taking it from basic science research are not necessary for doing most standard circuit analysis, to engineering of multi-qubit quantum systems capable unless mutual induction or dissipation is to be included, of preforming actual calculations [6–10]. In the mist of but can be very useful in several cases. A thorough under- this evolution a new discipline has emerged, coined quan- stand of the basic methods in Section I or experience in tum engineering, bridging the basic science of quantum superconducting circuit analysis is recommended as pre- mechanics with areas traditionally considered engineering requisite to this part. The methods presented in Section II fields [11]. It is expected that the advent of quantum en- are based on a method proposed by Ref. [58] which based gineering will lead to computational speed-ups, making it their method on electrical network graph theory [65]. In possible to solve classically un-solvable problems [12–14]. Section II we therefore also introduce some fundamental A particularly prominent platform for scalable quantum graph theory, and we include both mutual inductance and technology is superconducting circuits used for implement- dissipation in our equations of motion. ing qubits or even higher dimensional qudits. Compared In Section III we present examples of common super- to other quantum technology schemes, such as trapped conducting qubits, couplings, qudits, and more advanced ions [15–20], ultra cold atoms [21–25], electron spins in sil- circuits. Most of the examples are accessible after having icon [26–31] and quantum dots [32–36], nitrogen-vacancies read Section I. in diamonds [37, 38], or polarized photons [39–42], which Finally in Section IV we present an overview of the all encode in microscopic systems, methods and compare some of the more advanced topics such as ions, atoms, electrons, or photons, superconduct- to the basic methods. ing circuits are quite different as they are macroscopic in size and printed lithographically on plates much similar to classical computer chips [43–47]. The fact that these I. BASICS system exhibit microscopic behavior, i.e., quantum me- chanical effects, while being macroscopic in size has led to the notion of mesoscopic physics in order to describe In this section we present the basic tools needed for an- this intermediate scale [48–50]. A mesoscopic advantage alyzing superconducting circuits, starting from a lumped of superconducting circuits is the fact that microscopic circuit model to obtaining the Hamiltonian of the system, features such as energy spectra, coupling strengths, and quantizing it, removing the least contributing terms, and coherence rates depends on macroscopic circuit parame- finally truncating it at a desired number of quantum levels, ters. This means that one can design circuits such that i.e., a specific number of degrees of freedom. The tools the properties of the resulting quantum mechanical sys- presented in this section should be sufficient for analyzing tem, sometimes called an artificial atom [51–55], can be most superconducting circuit diagrams without including more or less tailor made to exhibit predictable behavior. mutual induction and dissipation. For these cases see Section II. Finally in Section I H we give an overview of In this tutorial we aim to give an introduction to the the basic methods used for analyzing superconducting field of superconducting circuit analysis for researcher circuits. The methods presented here can be considered new to the field. With this we aim to give the tools ’need to know’ when analyzing electrical circuits, and needed for tailoring macroscopic circuit to a desired quan- newcomers should in general read this entire section. tum mechanical behavior. The present tutorial can be viewed as an introduction to more advanced reviews of the field such as Refs. [11, 50, 51, 54, 56–64] and is by no means a review of current state-of-the-art technology A. Analyzing lumped-element circuit diagrams or practices, but rather a detailed introduction to the methods needed to analyse superconducting circuits in a In this section we will put forward an algorithmic ap- quantum setting. We will not discuss actual experimen- proach, or ’recipe’, to analyzing a lumped-element circuit tal production of superconducting circuits, but limit the diagram. The goal is to provide the reader with rigorous tutorial to theoretical analysis of such circuits. The tuto- methods to analyze and find the Hamiltonian of most rial assumes knowledge of undergraduate level quantum designs that may come up in the context of superconduct- mechanics, electrodynamics, and analytical mechanics, ing circuits. We will start by introducing the dynamical meaning that the tutorial should be accessible to readers variables used when analyzing superconducting circuits with a bachelor’s degree in physics or equivalent. and then present the basic elements of the circuits. The tutorial is organized as follows: In Section I we Our analysis takes its starting point in the lumped- present the basic knowledge needed in in order to analyze element model. This model simplifies the description of a superconducting circuits. The classical part of the analy- spatially distributed system (in our case a superconduct- sis revolves around the method of nodes for finding the ing electrical circuit) into a topology of discrete entities. Hamiltonian of the circuit, which is then subject to quan- We make the assumption that the attributes of the circuit tization. The methods presented in this section should (capacitance, inductance, and resistance) are idealized be enough for analyzing most standard superconducting into electrical components (capacitors, inductors, and re- circuits. sistors) joined by a network of perfectly conducting wires. 3

An example of a lumped circuit can be seen in Fig. 1. We Capacitor discuss the different components in Section I A 2. We assume all the circuits discussed in this tutorial to Inductor External be superconducting, meaning that there is no electrical re- Node sistance in the circuit and magnetic flux field are expelled flux from the wires (the Meissner effect). Therefore we will Branch ignore the dissipation in this analysis, however, we discuss how to add dissipative elements in the advanced section Josephson junction using the Caldeira-Leggett model [66], see Section II C. Ground node Ignoring dissipative elements effectively means that our circuits become lossless. While this does indeed make the analysis easier, even the best superconducting circuits experience external loss. Thus in a realistic description Figure 1. Example lumped circuit. A Josephson junction and noise should always be added. However, as these correc- a capacitor in parallel connected, by an inductor, to another pair of a Josephson junction and a capacitor. Such a pair of tions are often small they can be added as corrections in Josephson junction and capacitor is considered a transmon-like the end, e.g., when simulating the behavior of the circuit qubit. using an open quantum system approach.

are less degrees of freedom in the circuit than there are 1. Circuit variables branches in the circuit, these are, just as the currents and voltages, not completely independent but related through Circuit analysis aims to find the equations of motion Kirchhoff’s laws of an electrical circuit. Typically this means determining X the current and voltage through all components of the Qb = qn, (3a) all b arriving circuit. For simplicity, we only consider circuit networks at n containing two-terminal components, i.e., components X ˜ connected to two wires. Each such component is said to Φb = Φl, (3b) lie on a branch b and is characterized by two variables all b around l at any given time t: The voltage Vb(t) across it and the where qn is the charge accumulated at node n and Φ˜ l is current Ib(t) through it. We define the orientation of the the external magnetic flux through the loop l. A node can voltage to be opposite the direction of the current. Thus be understood as a point in-between elements/branches, these two are defined by the underlying electromagnetic see Fig. 1 where we denote nodes with a dot. A more field by rigorous definition of branches and nodes, using graph Z end of b theory, is given in Section II A. Vb(t) = E(t) d`, (1a) We have to eliminate superfluous degrees of freedom. · start of b In standard circuit theory there are two methods for 1 I Ib(t) = B(t) d`, (1b) this: The method of loops and the method of nodes. µ0 b · The method of loops (or mesh current method as it is commonly known in planar circuits) is usually taught in where µ0 is the vacuum permeability, and E and B are the electric field inside the wire and the magnetic field undergraduate courses on electrodynamics and we will outside the wire respectively. The closed loop in the therefore not discuss this method. Instead we will discuss second integral is done in vacuum encircling the given the, for our purpose, more useful method of nodes in element. As we use the lumped-element model, the voltage Section I B. A more advanced approach suitable for larger and current are independent of the integration path. Thus and more complex circuits can be found Section II B. we can choose the path such that the magnetic field is zero when integrating over the electric field and vice versa. For more details on integration of electromagnetic fields 2. Circuit components see e.g., Ref. [67]. We define the branch flux and branch charge variables Before elaborating on the method of nodes we discuss as the basic components of the circuit. Besides linear ele- ments we will consider a type of non-linear inductor called Z t 0 0 Josephson junctions which only occurs in superconducting Φb(t) = Vb(t ) dt , (2a) −∞ circuits. In an ideal superconducting circuit there is of Z t course no resistance, and dissipative elements will there- 0 0 Qb(t) = Ib(t ) dt , (2b) fore play a minor role when analyzing superconductive −∞ circuits. To summarize, we consider the following two- where it has been assumed that the system is at rest terminal elements: Linear capacitors, linear inductors, at t0 = with zero voltages and currents. As there Josephson junctions, and sources. −∞ 4

a. Capacitors The first component we consider is Ic the capacitor. For a general capacitor, the charge over V C L V the capacitor is determined as a function of the voltage c L q(t) = f(V (t)). In this tutorial we will only consider linear capacitors. For a linear capacitor the voltage is IL proportional to the charge stored on the capacitor plates Figure 2. Simple LC-oscillator circuit. A capacitor with q(t) V (t) = , (4) capacitance C is connected in a closed circuit with an inductor C of inductance L. The voltage over the two components are VC and VL respectively, while the current is IC and IL respectively. where C is the capacitance of the capacitor. This linear The resulting equations of motion is a harmonic oscillator. relationship is the defining property of the linear capacitor. In the real world this is merely an approximation, as there are small non-linear effects, which makes C a function of q VC = VL. Using Eqs. (2a), (2b), (5), and (8) we can set and V . These effects are usually small and therefore it is up the− equations of motion for the system standard to neglect them. Equation (4) can be rewritten 1 to the flux-charge relation using Eq. (2a) as Φ(¨ t) = Φ(t), (10) −LC q(t) Φ(˙ t) = V (t) = , (5) where we have introduced Φ(t) = ΦC (t) = ΦL(t) to get C rid of the subscripts. Evidently, the system− behaves as a where the dot indicates differentiation w.r.t. t. The simple harmonic oscillator in the flux. This is analogous current q(t) is equal to the branch current and using to a spring, where the flux is the position and the mass Eq. (2b) we find the branch current and spring constant are replaced by the capacitance and inverse inductance, respectively. I(t) = CΦ(¨ t). (6) c. Josephson junctions So far we have only consid- ered elements with linear current-voltage relations. For The energy stored in the capacitor is found by integrating reasons that will become clear when we treat the quan- the power P = V (t)I(t) from t = to t tization of lumped circuits, constructing a qubit from −∞ only linear elements is in no way straightforward. We 1 E = CΦ˙ 2(t). (7) therefore need non-linear elements which comes in the 2 form of the Josephson junction. The Josephson junction plays a special role in superconducting circuits as it has For superconducting circuits, typical values of the capaci- no simple analog in a non-superconducting circuit, due to tances are of the order 10 fF. In lumped-circuit diagrams the fact that it is related to charge quantization effects we denote the capacitor as a pair of parallel lines, see that occur in superconductors. Before elaborating on Fig. 1. the Josephson junction we give a short introduction to b. Inductors The time dependent current flowing . For a more in-depth discussion see through a general inductor is a function of the flux through e.g., Ref. [68]. it, I(t) = f(Φ(t)). We consider both linear inductors and In some materials the resistivity suddenly drops to a type of non-linear inductors called Josephson junctions. zero at low enough temperatures. This is a phase of For a linear inductor the current is proportional to the the material called superconductivity, which have some magnetic flux, exceptional properties. Together with the Meissner ef- 1 fect, i.e., that the material perfectly expels all magnetic I(t) =q ˙(t) = Φ(t), (8) L fields, the perfect conduction is the defining property of a superconductor. where L is the inductance of the inductor. Integrating The phase transition between the non-superconducting over the power again, the energy stored in an inductor is phase and the superconducting phase of a material hap- then pens because the material condenses into a so-called BCS 1 ground state. A priori this might seem impossible for E = Φ2(t). (9) electrons to condense in to a single quantum state due to 2L the Pauli exclusion principle. However, as L. N. Cooper For superconducting qubits, typical values of linear induc- suggested, some attractive force between the electrons tances are of the order 1 nH. In lumped-circuit diagrams lead to formation of electron pairs [69], which have integer we denote the inductor as a helix, see Fig. 1. spin and thus behave like bosons. This makes it possible As a short clarifying example we will consider the clas- for all these so-called Cooper pairs to condense into a sical LC-oscillator shown in Fig. 2. From Kirchhoff’s single quantum ground state and in this state the solid current law in Eq. (3a) we know that IC = IL, where becomes superconducting. IC and IL is the current through the capacitor and in- A Josephson junction consists of two superconduct- ductor respectively, and Kirchhoff’s voltage law gives us ing islands separated by a thin insulator, a non- 5

It is conventional to simplify notation in a way such that charge and fluxes becomes dimensionless, This is Cooper pair e- e- Tunneling done by using units where Superconducting Superconducting Φ0 ~ = 2e = 1 and thus = 1. (16) Insulator ~ 30 Å 2π

Figure 3. Sketch of a Josephson junction. Two superconduct- This means that we get rid of the cumbersome factor of ing materials separated by a thin insulator with a thickness 2π/Φ0 in the sinusoidal Josephson junction terms. Note in the order of 30 A. If a non-superconducting metal is used that in this notation the units of capacitance and induc- as a separator it can be several micrometers wide. Cooper tance become the inverse of an energy unit. Also, with pairs in the can then tunnel back and fourth between the two this choice of units the junction phase differences is equal superconducting materials. to the generalized flux φ = Φ. d. Voltage and current sources We can also treat con- stant voltage and current sources by representing them superconducting metal, or a narrow superconducting wire. as capacitors or inductors. Consider a constant voltage Cooper-pairs can then tunnel through the barrier from source V . This can be represented by a diverging capac- one island to the other, a phenomenon known as the itor, with a capacitance C , in which an initially Josephson effect [70, 71], see Fig. 3 for a sketch. The large charge Q is stored such→ that ∞ V = Q/C. tunneling rate (i.e., current) and voltage between the two In a similar manner a constant current source can islands depends on the superconducting phase difference, be represented be a diverging inductor, with inductance φ, between the islands through [72] L , in which an initially large flux Φ is stored, such that→I ∞= Φ/L. For examples of the use of constant voltage I(t) = Ic sin(φ(t)), (11) and current source see Sections III A and III B 1. V (t) = ~ φ,˙ (12) 2e B. Method of nodes where Ic is the critical current of the junction, which depends on the geometry of the junction. Equation (12) In this section we present the method of nodes for re- allows us to express the junction phase difference to the moving superfluous degrees of freedom and obtaining the generalized flux through Φ = ~φ/2e. The charge and flux Hamiltonian and Lagrangian. This method solves most are thus related through practical problem involving Josephson junction not includ- ing mutual inductance and dissipation. This discussion  Φ(t) q˙(t) = Ic sin 2π , (13) follows the method proposed by Devoret [50, 73]. Φ0 First we define an active node as a node connected to more than one type of circuit element. Contrary to the where we have defined the magnetic flux quantum Φ0 = active node is the passive node, which is a node where h/2e. The Josephson junction thus work as a flux depen- only one type of element converge. We further consider dent inductor with inductance given by [57] nodes which lie between an element and ground. These nodes are inactive since the flux through them is zero  ∂I −1 L L(Φ) = = J , (14) and thus they do not contribute to the dynamics of the ∂Φ  Φ  system. We denote these ground nodes. Considering the cos 2π Φ0 example circuit in Fig. 1 we note that the two nodes on either side of the inductor are active nodes, while the where we have defined the Josephson inductance LJ = bottom node is the ground node. Φ0/2πIc. For superconducting qubits typical values of Consider now an electromagnetic circuit, say the exam- Josephson inductances are of the order 100 nH. The en- ple circuit in Fig. 1. We represent the circuit as a set of ergy of a Josephson junction is also non-linear nodes, i.e., the example circuit has three nodes, and a set of branches, which are equal to the set of elements in the 2    Φ0 1 Φ circuit, i.e., the example circuit has five branches, one for E = 2 1 cos 2π , (15) (2π) LJ − Φ0 each component. We now divide the system into its capacitive subnet- where we will often neglect the constant term. We denote work, containing only branches and nodes connected to the factor in front of the bracket as the Josephson energy capacitors, and its inductive subnetwork, which of course 2 2 of the Josephson junction, EJ = Φ0/(2π) LJ = Φ0Ic/2π. contains branches and nodes connected to inductors (i.e., In this tutorial we denote Josephson junctions as a box both linear and Josephson junctions). In the example with an x inside in lumped-circuit diagrams, see Fig. 1. circuit in Fig. 1 the two capacitor branches and all three Other literature might simply use an x without a box to nodes are in the capacitive subgraph, while the inductor denote the Josephson junction. branch and the two Josephson junction branches are in 6

(a) Table I. Energies for different elements either on the spanning tree or a closure branch of the circuit. The magnetic flux through the closure branch due to external fields is denoted Φ˜ b. Element Spanning tree Closure branch

Linear C 2 C ˙ 2 (φ˙n φ˙n0 ) (φ˙n φ˙n0 + Φ˜ b) (b) capacitor 2 − 2 − Linear 1 2 1 2 (φn φn0 ) (φn φn0 + Φ˜ b) inductor 2L − 2L − Josephson EJ cos (φn φn0 ) EJ cos(φn φn0 + Φ˜ b) junction − − − −

incoming and outgoing branch fluxes, with the correct sign depending on the direction of the flux. With this in Figure 4. Highlight of (a) the capacitive subgraph and (b) mind we can write the branch fluxes in terms of the node the inductive subgraph of the example circuit. fluxes

Φb∈E =φn φn0 , (17a) the inductive subgraph together with the three nodes. T − ˜ Note that nodes can be in both subgraphs at the same Φb∈E¯ =φn φn0 + Φ, (17b) T − time. See Fig. 4. 0 In a realistic circuit every node connected to an induc- where n and n are the nodes at the end and start of ˜ tance should also be connected to a capacitor, due to the given branch respectively and Φ is the external flux the fact that a parasitic capacitance will be present for through the element on the branch. Note that the external all inductors. It can, however, be advantageous if this flux only occurs if the branch is a closure branch (this is parasitic capacitance can be made as small as possible. due to Kirchhoff’s laws). Note that as we always consider The capacitive subnetwork consists only of linear capac- even powers of the flux it is often irrelevant which node itors, and thus we can express the energy of a capacitive is considered the start node and which is considered the branch in terms of the voltages, i.e., the derivative of the end node. The ground node has zero flux and is therefore flux, using Eq. (7). By doing this we have in fact now often not included in circuit diagrams. broken the symmetry between the charge and flux, and Substituting the node fluxes into the expressions for the flux can now be viewed as the “position”. With this the energy of the different components seen in Eqs. (7), treatment the capacitive energy becomes equivalent to (9), and (15) we can express the energies as a function of the kinetic energy, while the inductive energy becomes the node fluxes. The results can be seen in Table I. equivalent to the potential energy. Starting from the ground node we now construct a spanning tree, . This is constructed by connecting every 1. Equations of motion node to the groundT node by only one path along the tree (see Definition 3 in Section II A for a more mathematical The equations of motion can, in principle, now be found definition using graph theory). Note that there are often using Kirchhoff’s circuit laws for every node and every several choices for both ground node and spanning tree. loop in the circuit. This is, however, often quite cum- This is not a problem for the analysis and can be seen bersome and it is usually easier to find the Lagrangian analogous to the choice of a particular gauge in electro- instead. The Lagrangian is obtained by subtracting the magnetic field theory or the choice of a coordinate system potential (inductive) energy term from the kinetic (capac- in classical mechanics. Note that even though we refer itive) energy term. to only a single node as the ground node, there can be Writing the Lagrangian up like this can be rather te- several nodes connected to ground in the circuit. A choice dious for larger circuits since it includes a lot of sums. of spanning tree in the example circuit could be the ca- We therefore seek a more elegant way to write the La- pacitive subgraph as seen in Fig. 4(a), however, there are grangian; this is achieved using matrix notation. First we several other valid choices. list all the nodes 1 to N and define a flux column vector T We have now partitioned the elements into two sets: φ = (φ1, . . . , φN ), where T indicates the transpose of The set of elements on the spanning tree, ET and its the vector. Note that we do not include the ground node complementary set, E¯T = E ET , i.e., the elements not since it equals zero, and, as we will see later, it does not on the spanning tree. We call\ the latter set the set of contribute to the true degrees of freedom in any case. In closure branches due to the fact that its elements closes the case of the example circuit in Fig. 1 the flux column T the loop of the spanning tree. vector is φ = (φ1, φ2). As the circuit is symmetric it The flux φn of node n can be written as a sum of does not matter which node corresponds to 1 and 2. For 7 the sake of concreteness we label the right node 1 and the If we consider the example circuit again, the inductive left node 2. matrix is simply We are now ready to set up the capacitive matrix C of   1/L12 1/L12 the system. The non-diagonal matrix elements are simply L−1 = − . (22) 1/L12 1/L12 minus the capacitance Cjk connecting nodes j and k. − The diagonal elements consist− of the sum of values in the With this the inductive energy of the example circuit corresponding row or column, multiplied by minus one. becomes If a node is connected to ground via a capacitor, this capacitance must also be added to the diagonal element. 1 T −1 1 Einductors = φ L φ + (φ1 φ2)Φ˜, (23) With this N N matrix we can write the kinetic energy 2 L12 − term as × where Φ˜ is the external flux through the circuit loop. 1 ˙ T ˙ When there is only one (or few) inductors, as in the Ekin = φ Cφ, (18) 2 example circuit, it might be more straightforward to write the energy without the matrix notation. For the example where we have assumed that all capacitors are placed circuit it simply becomes on the spanning tree or that the external flux is time- independent. If this is not the case one must add a term 1 2 Einductors = (φ1 φ2 + Φ)˜ . (24) similar to the one in Eq. (21). 2L12 − In the case of the example circuit the capacitive matrix becomes The Lagrangian can now be obtained simply by sub- tracting the inductive energies from the capacitive energies   C1 0 C = . (19) = Ekin Epot = Ekin Einductors EJJ, (25) 0 C2 L − − − where EJJ is the sum of energies from the Josephson Had the linear inductor in between the two node been junction, which we do not write in matrix form as it a capacitor, the capacitive matrix would have taken the includes cosines. form The full Lagrangian of the example circuit is thus   C1 + C12 C12 1 1 C = − . (20) = φ˙ T Cφ˙ (φ φ + Φ˜ )2 C12 C2 + C12 1 2 0 − L 2 − 2L12 − (26) ˜ ˜ We now consider the contribution from the linear in- + EJ,1 cos(φ1 + Φ1) + EJ,2 cos(φ2 + Φ2), ductors. We set up the inductive matrix L−1 in the same where Φ˜ are the external fluxes through the different way as the capacitive matrix. The non-diagonal elements i loops. With the Lagrangian, one can obtain the equations are 1/L connecting nodes j and k, while the diagonal jk of motion from Lagrange’s equation elements− consist of the sum of values in the correspond- ing row or column, multiplied by minus one. If a node d ∂ ∂ is connected to ground via an inductor, this inductance L = L . (27) dt ∂φ˙ ∂φn must also be added to the diagonal element. Of course, n if no inductor is connecting two node, the non-diagonal Applying this to the example circuit we find the equations element should be zero. Contrary to capacitors, inductors of motion are rarely placed on the spanning tree, but on closure ¨ 1 EJ,n branches, and we must therefore remember to include φn = (φ1 φ2 + Φ˜ 0) sin(φn + Φ˜ n). (28) external magnetic flux in the term. Thus the energy due ∓L12Cn − − Cn to linear inductors becomes where the minus is for n = 1 and the plus is for n = 2.

1 T −1 X 1 Einductors = φ L φ + (φn φn0 )Φ˜ b, (21) 2 Lb − b∈E¯T 2. Obtaining the Hamiltonian where we have removed all irrelevant constant terms. The The Hamiltonian of the circuit can be found by a simple second term sums over all the inductive branches of the Legendre transformation of the Lagrangian. First we 0 circuit where n and n are the nodes connected by branch define the conjugate momentum to the node flux by b. If there is no inductor the term is zero. This term can be relevant for a range of phenomena involving Josephson ∂ q = , (29) n L˙ junctions. Note that in case there are capacitors on closure ∂φn branches, a term similar to the second term in Eq. (21) must be added to Eq. (18) for these, unless the magnetic which in vector form becomes q = Cφ˙. If the capacitance flux is constant in time. matrix is invertible we can express φ˙ as a function of q, 8 which we denote node charges, due to the fact that they This is useful for implementing a system with direct and corresponds to the algebraic sum of the charges on the thus strong multi-body interactions [75]. For example capacitances connected to node n. three-body couplings are necessary in gauge theories, and The Hamiltonian can now be expressed in terms of the four-body couplings are necessary for quantum annealing. node charges qn for the kinetic energy and node fluxes for There are as such many proposals for their implementa- the potential energy tion, though these are usually indirect, i.e., the interaction is an effective interaction and does not actually occur in =φ˙ T q the Hamiltonian. Indirect interactions are generally slower H − L 1 (30) and noisier, as they consist of multiple subprocesses. Indi- = qT C−1q + E (φ), 2 pot rect interactions can be implemented using perturbation theory [76–79], mutual inductive couplings to a common where the potential energy is a non-linear function of resonator [80–83] and the decomposition of the multi- the node fluxes. The independent variables corresponds body interaction to many two-body interactions involving to degrees of freedom, while the non-independent corre- ancilla qubits [80, 84–88], among other approaches. sponds to offset charges. Note that the functional form of the Hamiltonian may differ depending on the choice of spanning tree, however, it always returns the same total 4. Change of basis energy. With the Hamiltonian it is possible to find the equations Here we present a method for changing into the normal of motion using Hamilton’s equations mode basis of a circuit. Given a circuit with N nodes 2 −1 −1 ∂ ∂ and a matrix product Ω = C L let v1, v2,..., vn ˙ 2 φn = H , q˙n = H , (31) be the orthogonal eigenvectors of Ω , with eigenvalues ∂qn −∂φn λ1, λ2, . . . , λn. Let φ be the usual vector of the node which is of course equivalent to Lagrange’s equations fluxes of the circuit. We can then introduce the normal Eq. (27). modes ψ via φ = Vψ, (34) 3. Normal modes where

Lagrange’s equations tell us that q˙n = 0 for all passive   | | | nodes. This means that the circuit has at most the same V = v1 v2 vN  , (35) number of true degrees of freedom as the number of ··· active nodes except the ground node. The number of true | | | degrees of freedom is identical to the number of normal is a matrix whose columns are the eigenvectors of Ω2. 2 modes of the system. If all inductors can be approximated Notice that ψi = vi φ/ vi , such that the eigenvectors · | | as linear inductors (and external fluxes are ignored), the vi are exactly the normal modes in terms of the node Lagrangian takes the form fluxes up to normalization. The kinetic energy term in Eq. (18) can now be written 1 1 = φ˙ T Cφ˙ φT L−1φ. (32) L 2 − 2 1 ˙ T ˙ Ekin = ψ Kψ, (36) This particular nice form of the Hamiltonian means that 2 the equations of motion takes the form where we have introduced the capacitance matrix in the transformed coordinates K = VT CV. Note that we can Cφ¨ = L−1φ, (33) − perform a similar transformation for any representing V which is essentially Hooke’s law in matrix form where an ortonormal transformation of the basis. −1 the capacitances plays the role of the masses and induc- If C and L can be simultaneous diagonalized then 2 K a diagonal matrix with entries λi vi . In terms of the tive matrix plays the role of the spring constants [74]. | | The normal modes of the full systems can be found as canonical momenta p conjugate to ψ, the kinetic energy the eigenvectors of the matrix product Ω2 = C−1L−1 takes the usual form associated with non-zero eigenvalues. These non-zero 1 E = pT K−1p, (37) eigenvalues correspond to normal mode frequencies of the kin 2 circuit squared. Note that if C and L−1 can be diagonal- ized simultaneously, their eigenmodes become the normal where the inverse of K is trivial to find, as it is also a 2 modes of the system. diagonal matrix, whose entries are 1/(λi vi ) (below we | | It can be advantageous to find these eigenmodes and consider the case λi = 0, but we ignore it for now). change basis into them (see Section I B 4), as this can We must also consider how the higher order terms of be used to control unwanted couplings between modes. the inductor behave under this coordinate transformation. 9

Even though we have approximated all inductors as linear tum, qˆn operator and the flux operator do not commute in order to find the normal modes, higher order terms from if they corresponds to the same node. Instead they obey the Josephson junctions still contribute as corrections, the canonical commutator relation often leading to couplings between the modes. Such ˆ ˆ ˆ terms transform the following way [φn, qˆm] = φnqˆm qˆmφn = i~δnm, (40) − X φk φl [(vi)k (vi)l] ψi, (38) where δnm is the Kronecker delta. This relation is only − → − i valid if the electric state of the nth node is a true degree of freedom, meaning that neither φˆ or qˆ are constants where (v ) is the kth entry of v . Such fourth order n m i k i of motion, i.e., Hamilton’s equations in Eq. (31) does not terms can result in both two-body interactions as well yield zero. as more-than-two-body interactions. These multi-body We could also have used the same approach as Dirac, interactions can complicate the equations of motion more and used that the value of the classical Poisson bracket that the change of basis simplifies them. Coordinate determines the value of the corresponding commutator up transformations are therefore often most useful where the to a factor of i [89]. Using this approach for the branch capacitors are symmetrically distributed, which results in ~ flux operator and the charge operator we find that the simple normal modes. Poisson bracket is The center of mass (CM) mode plays a special role in an-   alytical mechanics as it often decouples from the dynamics X ∂Φb ∂Qb ∂Qb ∂Φb Φ ,Q = = 1, (41) of the system. The same is the case for electrical circuits. b b ∂φ ∂q ∂φ ∂q T { } n n n − n n ± The center of mass mode has vcm = (1, 1,..., 1) , which 1 yields ψCM = 2 (φ1 +φ2 + +φN ). This mode is always N ··· where the sign is plus for a capacitive branch and minus present and it corresponds to charge flowing equally into for an inductive branch. Following Dirac’s approach we every node of the circuit from the ground and oscillating arrive at the following commutator relation back and forth between ground and the nodes. Further- ˆ ˆ more, since all its entries are identical it always disappears [Φb, Qb] = i~, (42) in the linear combination of Eq. (38). Hence, this mode ± completely decouples from the dynamics, justifying calling Note that in general these branch operators are not con- it the center-of-mass mode. jugate in the Hamiltonian. One must still find the true The decoupling of this mode is related to how we can degrees of freedom before quantization is applied. arbitrarily choose a node in our circuit as the ground node, whose node flux does not enter our equations, or rather is identically set to zero. This redundancy of the 2. Effective energies node fluxes, allowing for one degree of freedom to be ˆ removed, is what is surfacing here again. If the circuit Consider the generalized momentum qˆ = Cφ˙ . The time is not grounded, if a ground node has not been chosen, derivative of the generalized momentum is exactly the then λCM = 0, making K singular. We therefore always current through the capacitors, I = Cφ¨. Note that in the assume the circuit is grounded such that λCM = 0. 6 limiting case of one node, this reduces to the current over For an example of multi-body interactions see Sec- a single parallel-plate capacitor, as it should. This makes tion III F. For other examples of change of basis see Sec- sense since we can think of the conjugate momentum as tions III C 4 and III D 2. the sum of all charges on the capacitor. We therefore define

C. Quantization and effective energies qˆn nˆn = , (43) − 2e 1. Operators and commutators as the net number of Cooper pairs stored on the nth node. If we consider the kinetic energy of a circuit we can write After the Hamiltonian has been obtained classically we are now ready to transform to the quantum mechanical 1 e2 E = qˆT C−1qˆ = 4 nˆ T C−1nˆ. (44) description of the circuit. This is done through canonical kin 2 2 quantization, replacing all the variables and the Hamilto- 2 nian by operators Now for a diagonal element we have 4EC,nnn, where we have defined the effective capacitive energy of the nth ˆ φ φ, node as → q q,ˆ (39) 2 → e −1 ˆ. EC,n = (C )(n,n), (45) H → H 2 ˆ The node flux operators, φn, corresponding to position which is equivalent to the energy required to store a single coordinates, all commute. However, the conjugate momen- charge on the capacitor. Note that in our dimensionless 10 notation from Eq. (16) we have nˆn = qˆn while the we define the step operators effective energy becomes E = (C−1) − /8. C,n (n,n) 1  1  In a similar manner we will introduce the effective ˆb = φˆ ipζnˆ , (49a) √ √ζ − energies of the linear and Josephson inductances, EL,n 2 and E of each node. The effective linear inductive 1  1  C,n ˆb† = φˆ + ipζnˆ , (49b) energy is simply the diagonal elements of L, which is √2 √ζ equivalent to the sum of the inverse inductances of the inductors connected to the given node. The effective where we have defined the impedance Josephson energy is found the same way, it is the sum of s 4E Josephson energies connected to the given node. ζ = C . (50) Returning to our example circuit in Fig. 1, we can now EL + EJ /2 write it using operators and effective energies. It becomes The step operator fulfill the usual commutator relation ˆ 2 2 2 † =4 EC,1nˆ + EC,2nˆ + EL,12(φ1 φ2) [ˆb, ˆb ] = 1. (51) H 1 2 − (46) EJ,1 cos φ1 EJ,2 cos φ2, − − Expressing the flux and conjugate momentum operators in terms of the step operators, where the coupling energy of the linear inductor is EL,12 = r 1/2L and we have set Φ˜ = 0 for simplicity. The effective ζ 12 φˆ = (ˆb + ˆb†), (52a) energy of the Josephson junctions is simply the Josephson 2 energy. Note that since our example does not include i ˆ ˆ† any coupling capacitors we do not obtain any coupling nˆ = (b b ), (52b) −1 √2ζ − term (C )(1,2)nˆ1nˆ2 since C is diagonal. In reality this is rarely the case. we can rewrite the oscillator part of the Hamiltonian as s  1   1 ˆSHO = 4 EC EL + EJ Nˆ + , (53) D. Recasting to interacting harmonic oscillators H 2 2 where we have introduced the usual number operator If we require the effective capacitive (charging) energy, Nˆ = ˆb†ˆb. EC , to be much smaller than the effective Josephson en- Doing this we must of course also rewrite all quadratic ergy, EJ , the flux will be well-localized near the bottom of and quartic interaction terms. An overview of how to the potential (we omit the node subscript in this section rewrite these into step operators can be seen in Table II. for simplicity). This is equivalent to a heavy particle mov- Returning to our example circuit in Fig. 1 we can write ing near the equilibrium position. If we further require the Hamiltonian using step-operators the anharmonicity of effective potential (flux-dependent part of the Hamiltonian) of the superconducting circuit ˆ ˆ†ˆ ˆ†ˆ α1 ˆ† ˆ 4 =ω1b1b1 + ω2b2b2 + (b1 + b1) to be sufficiently large, we are in what is called the trans- H 12 (54) α2 ˆ† ˆ 4 ˆ† ˆ ˆ† ˆ mon regime, named after the transmon qubit [90]. In + (b2 + b2) + g12(b1 + b1)(b2 + b2), this case we can Taylor-expand the full potential part of 12 the Hamiltonian up to fourth order in φ, such that the where we have omitted all constant terms, which are Josephson junction terms takes the form irrelevant for the dynamics. We have further defined s 1 2 1 4  1  EJ cos φ EJ EJ φ + EJ φ . (47) ω =4 E E + E , (55a) ' − 2 24 n C,n L,12 2 J,n 2 Throwing away the irrelevant constant term, we are left ζn αn = EJ,n, (55b) with a Hamiltonian consisting of pure second and fourth − 8 order terms. If we require the couplings between differ- p g12 = ζ1ζ2EL,12, (55c) ent parts of the superconducting circuit to be small, we s− can treat each part individually as harmonic oscillators 4EC,n ζn = , (55d) perturbed by quadratic and quartic interactions (each EL,12 + EJ,n/2 part will then later become a qubit). For each part in our system we have a simple harmonic oscillator on the form where we will refer to ωn as the frequency, αn as the anharmonicity, and g12 the coupling of our (an)harmonic   2 1 ˆ2 oscillators. The reason for not including the factor of ˆSHO = 4EC nˆ + EL + EJ φ . (48) H 2 1/12 in α will become apparent in Section I E. Note that if the effective inductive energy is zero, EL,n = 0 then the The simple harmonic oscillator is well understood quan- anharmonicity in Eq. (55b) becomes αn = EC,n, which tum mechanically, and using the algebraic approach [91] holds generally. − 11

(or free) part and an interacting part ˆ = ˆ0 + ˆI,S. Table II. Overview of the different elements and the operators States in the interaction picture are definedH asH H they map to. Subscripts are included where appropriate, and refer to different nodes. All constant terms have been neglected. iHˆ0t ψ, t I = e ψ, t S, (57) The impedance factor can be seen in Eq. (50). | i | i Element Hamiltonian term Step operators where the subscript I refers to the interaction picture. q 2 ˆ2 1 ˆ†ˆ The operators in the interaction picture are defined as All termsn ˆ + φ 4 EC EL + 2 EJ b b Linear i † ˆ ˆ nˆ √ (ˆb ˆb ) ˆ iH0t ˆ −iH0t capacitors 2ζ − I = e Se , (58) 2 † 2 O O nˆ 1 (ˆb ˆb ) − 2ζ − ˆ 1 ˆ ˆ† ˆ ˆ† where S is an operator in the Schr¨odinger picture. nˆinˆj (bi bi )(bj bj ) − 2√ζiζj − − O q It is then possible to show the Schr¨odingerequation in Linear φˆ ζ (ˆb† + ˆb) the interaction picture becomes inductors 2 φˆ2 ζ (ˆb† + ˆb)2 2 ∂ ζ ζ i ψ, t = ˆ ψ, t , (59) ˆ ˆ √ i j ˆ† ˆ ˆ† ˆ I I I φiφj 2 (bi + bi)(bj + bj ) ∂t| i H | i 2 Josephson ˆ4 ζ ˆ† ˆ 4 ˆ ˆ φ (b + b) ˆ iH0t ˆ −iH0t junctions 4 where I = e I,Se is the interaction part of the ˆ3 ζ3/2 ˆ† ˆ 3 HamiltonianH in theH interaction picture. In general one can φ 23/2 (b + b) 3/2 1/2 ζ ζ transform to any rotating frame using the transformation ˆ3 ˆ i j ˆ† ˆ 3 ˆ† ˆ φi φj 4 (bi + bi) (bj + bj ) rule ˆ2 ˆ2 ζiζj ˆ† ˆ 2 ˆ† ˆ 2 φi φj 4 (bi + bi) (bj + bj ) ˆ † † d (t) ˆ ˆR = ˆ(t) ˆ ˆ(t) + i U ˆ(t) (60) H → H U H U dt U E. Time averaged dynamics where ˆ(t) = exp( i ˆ0t) is the unitary transformation of U − H the rotating frame ˆ0 one wishes to transform into. This When analyzing the Hamiltonian of the circuit it is transformation ruleH holds for any unitary transformation often advantageous to consider which terms dominate the as is quite useful to keep in mind. The advantages of time evolution and which terms only give rise to minor rotating in to a specific frame is that we can highlight corrections. The latter can often be neglected, without some desired physics while ignoring other parts that are changing the overall behavior of the system. It can of- well understood. This is analogous to choosing a refer- ten be difficult to determine which terms dominate as ence frame rotating with the Earth when doing classical different scales influence the dynamics of the system, this physics in a reference frame fixed on the surface of the stems from the fact that the modes of the oscillators are Earth. Note that Eq. (60) is equivalent to transforming usually of the order GHz while the interactions between the Hamiltonian into the interaction picture ˆ ˆI the different oscillators are usually much smaller, of the H → H when ˆ0 is the non-interacting part of the Hamiltonian, order MHz. We therefore employ separation of scales, as theH second term simply removes the non-interacting such that we remove the large energy differences of the part of the Hamiltonian. modes from the Hamiltonian making it possible to see One can also show that the time evolution in the inter- the finer details of the interactions. In order to do this action picture is governed by a Heisenberg equations of we first introduce the concept of the interaction picture motion where the interacting part of the Hamiltonian is in focus d ˆI = i[ ˆI , ˆ0], (61) dtO O H 1. Interaction picture where we have assumed no explicit time dependence in ˆS. Note that this implies that the voltage of the bth Consider the state ψ, t S at time t which satisfies the O ˆ ˆ ˆ | i branch can be calculated as Vb = i[ , Φb]. For more Schr¨odingerequation information about the interaction pictureH see e.g., Ref. [91]. ∂ i ψ, t S = ˆ ψ, t S, (56) ∂t| i H| i 2. Rotating wave approximation where ˆ is the Hamiltonian, and the subscript S refers to theH Schr¨odingerpicture. In the Schr¨odingerpicture operators are time independent and state are time de- Consider now the weakly anharmonic oscillator as seen pendent. We wish to change into the interaction picture in Fig. 5(c), which has the quantized Hamiltonian where both the operators and state are time dependent. α ˆ = ωˆb†ˆb + (ˆb† + ˆb)4, (62) We therefore split the Hamiltonian into a non-interacting H 12 12 where we have removed all constant terms. We have that such that the dressed frequency becomes ω˜ = ω + α = ˆ†ˆ†ˆˆ the frequency is ω = √8EC EJ and the anharmonicity is √8EC EJ EC . The remaining (b b bb)-term makes the − α = EC where EC and EJ is the effective capacitive oscillator anharmonic. For this reason we call α the anhar- energy− and Josephson energy respectively. Now we choose monicity of the anharmonic oscillator. Thus if we remove ˆ the first term as the non-interacting Hamiltonian, 0 = terms that are not conserving energy the Hamiltonian ωˆb†ˆb. We now want to figure out how the step-operatorsH takes the form (in the Schr¨odingerpicture) behave in the interaction picture, i.e., we want to calculate α Eq. (58) for the step-operators. First we notice that ˆ =ω ˜ˆb†ˆb + ˆb†ˆb†ˆbˆb. (65) nˆ† ˆ† n H 2 ˆ b = b ( ˆ0 + ω) . Using this and expanding the H0 H exponential functions we can prove that Consider now an interaction term like the last term of ˆ ˆ eiH0tˆb†e−iH0t = ˆb†eiωt. (63) Eq. (54) By taking the complex conjugate we find a similar expres- ˆ† ˆ† ˆ† ˆ† ˆ†ˆ ˆ†ˆ ˆ†ˆ† ˆ ˆ (bi + bi )(bj + bj) = bi bj + bi bj + bi bj + bibj. (66) sion for ˆb, but with a minus in the exponential factor. We now wish to consider how different terms of the step- Changing into the interaction picture we realize that operators transform into the interaction picture. Starting the two last terms obtain a phase of exp i(ωi + ωj)t, with the number operator Nˆ = ˆb†ˆb, we see that it does which can be considered a fast oscillating± term if the † not change as the exponential factor from ˆb cancels the frequencies ωi + ωj are much larger than the interaction exponential factor from ˆb. This is not surprising as the strength, which is usually the case. We can therefore non-interacting Hamiltonian is chosen exactly as the num- safely neglect these non-conserving terms. The two first ˆ†ˆ† terms on the other hand obtains a phase of exp( iδt), ber operator. However, if we consider terms like Jb b ± where δ = ωi ωj is the detuning of the two oscillators. we find that in the interaction picture they take the form − Jˆb†ˆb†e2iωt. If ω is sufficiently large compared to the fac- It is therefore tempting to say that these terms only contribute if ωi ωj. This is, however, not the whole tor, J, in front of the term (which is often the case in ≈ superconducting circuit Hamiltonians where ω GHz, story, in fact we find that while other terms are usually of the size J MHz∼), these ˆ†ˆ iδt ˆ ˆ† −iδt ˆ†ˆ ˆ ˆ† terms will oscillate very rapidly on the timescale∼ of J. The bi bje + bibje = (bi bj + bibj) cos δt (67) time average over such terms is zero, and we can therefore ˆ†ˆ ˆ ˆ† + i(b bj bib ) sin δt, neglect them as they only give rise to minor corrections. i − j This is essentially the rotating wave approximation (RWA) which can be useful in some situations, e.g., when driving which is thoroughly used in atomic physics [92, 93]. The qubits, see Section I G. However, as a general rule-of- ˆˆ story is the same for bb terms. In fact all terms which do thumb one can neglect these terms unless ωi ωj. not conserve the energy of the system, i.e., terms where For a more general discussion on the validity≈ of the the number of ˆb†-operators are not equal to the number time averaging dynamics see Ref. [94]. of ˆb-operators, will rotate rapidly and can therefore safely If we consider the example circuit in Fig. 1 under the be neglected. Note that in spite of the fact that we refer assumption that αn ωn, we time average its Hamil- to this as energy conserving and non-conserving terms, tonian in Eq. (54).| We|  chose the non-interacting Hamil- ˆ†ˆ ˆ†ˆ we could just as well talk about conservation of quanta, tonian as 0 = ω1b1b1 + ω2b2b2, which means that the as created and destroyed by the step operators, as is im- interactingH part of the Hamiltonian in Eq. (54) becomes plied from the discussion above. It is important to point out that in spite of the naming, the ’conservation’ is not α1 ˆ†ˆ†ˆ ˆ ˆ†ˆ α2 ˆ†ˆ†ˆ ˆ ˆ†ˆ ˆI = (b b b1b1 + b b1) + (b b b2b2 + b b2) related to a conservation law resulting from a symmetry, H 2 1 1 1 2 2 2 2 i.e. like in Noether theorem. Rather, the statement here ˆ†ˆ iδt ˆ ˆ† −iδt + g12 b b2e + b1b e (68) means that the energy conserving terms are much more 1 2  important than the non-conserving terms as long as the ˆ ˆ −i(ω1+ω2)t ˆ†ˆ† i(ω1+ω2)t + b1b2e + b1b2e , conditions for using the rotating wave approximation are satisfied. where we have defined the detuning δ = ω1 ω2. Assuming Now consider the anharmonicity term of Eq. (62). − ω1 ω2 we have that δ 0 and if we further assume that When only including energy conserving terms and remov- ' ' ωn g12 then the last two terns are fast oscillating and ing irrelevant constants, the anharmonicity term takes can thus be neglected. We are usually interested in the the form Hamiltonian in the Schr¨odinger picture, and we write the α 1  Hamiltonian in this frame. In this frame the fast rotating (ˆb† + ˆb)4 =α ˆb†ˆb†ˆbˆb + ˆb†ˆb 12 2 (64) terms are denoted non-conserving terms, as they do not + Non-conserving terms. conserve the energy (in the sense discussed above), and using we interaction picture we have showed that they The last term, ˆb†ˆb, is simply the number operator, and only contribute small corrections. This means that we can we can therefore consider it a correction to the frequency, write the Hamiltonian in the Schr¨odingerpicture without 13

(a) (c) divided by the first energy gap α α = E E , α = . (70) E 12 01 r CL C J − E01 Note that this anharmonicity is the same anharmonicity in front of the (ˆb†ˆb†ˆbˆb)-terms as mentioned in previous sections. As a rule-of-thumb the relative anharmonicity (b) (d) should be at least a couple of percent in order to make an effective qubit. It does not matter whether the an- harmonicity is positive or negative. For transmon-type qubits it will be negative, while it can be either positive or negative for flux-type qubits. The relative anharmonicity p is proportional to EC /EJ which means that this ratio must be of a certain size for the anharmonicity to have an effect. This is in contrast to what was discussed in the beginning of Section I D where we argued that we required this ratio to be as low as possible in order to allow for the expansion of cosines. Thus we need to find a suitable regime for the ratio, EC /EJ . This regime is usually called the transmon regime. Figure 5. (a) Circuit of an LC-oscillator with inductance L and When the anharmonicity is sufficiently large we can capacitance C. We denote the phase on the superconducting distinguish between the energy gaps. This means that a island φ, while the ground node has phase zero. (b) Energy transition between the ground state and the first excited potential of a quantum harmonic oscillator, as can be obtained state does not require the same amount of energy as a by an LC-circuit. Here the energy levels are equidistantly transition between the first and second excited states, spaced ~ω apart. (c) Josephson junction qubit circuit, where the linear inductor has been replaced by a non-linear Josephson or any other pair of states for that matter. This is due junction of energy EJ . (d) The Josephson junction changes to the fact that the anharmonicity term varies in size the harmonic potential (blue dashed) into a sinusoidal potential for different energy levels as we will see in the following (orange solid), yielding non-equidistant energy levels. sections. In the following we assume that we have a suffi- ciently large anharmonicity and truncate the system in to a two-level system, however, there is nothing stopping non-conserving terms as us from keeping more levels.

ˆ†ˆ ˆ†ˆ α1 ˆ†ˆ†ˆ ˆ ˆ =˜ω1b b1 +ω ˜2b b2 + b b b1b1 H 1 2 2 1 1 α (69) 1. Two-level model + 2 ˆb†ˆb†ˆb ˆb + g (ˆb†ˆb + ˆb ˆb†), 2 2 2 2 2 12 1 2 1 2 In a two-level system we can represent our states as where we have introduced the dressed frequencies ω˜n = ω + α . 1 0 n n 0 , 1 (71) | i ∼ 0 | i ∼ 1

F. Truncation In this reduced Hilbert space all operators can be ex- pressed by the Pauli matrices, A harmonic oscillator, as one gets from a regular LC- 0 1 0 i 1 0  circuit, has an infinite amount of states [see Fig. 5(b)]. σx = , σy = , σz = , (72) 1 0 i −0 0 1 This is not desirable as we wish to only consider the − lowest states of the system. However, when we intro- and the identity. If we view the unitary operations as duce a Josephson junction instead of a linear inductor, we rotations in the Hilbert space we can parameterize the introduce an anharmonicity, see Fig. 5(a) and (c). The an- superposition of the two states using a complex phase, φ, ˆ†ˆ†ˆˆ harmonicity stems from the (b b bb)-terms (see Eq. (65)) and a mixing angle, θ and can be viewed as perturbations to the harmonic os- cillator Hamiltonian, if α ω. This anharmonicity θ  θ  | |  ψ = cos 0 + sin eiφ 1 , (73) changes the spacing between the energy levels of the har- | i 2 | i 2 | i monic oscillator, making it an anharmonic oscillator [see Fig. 5(d)]. For this reason we require the anharmonicity where 0 θ π and 0 φ < 2π. With this we can to be of a certain size. The anharmonicity is defined as the illustrate≤ the qubit≤ as a unit≤ vector on the Bloch sphere, difference between the first and second energy gap, while see Fig. 6. Unitary operations can then be seen as rota- we define the relative anharmonicity as the anharmonicity tions on the sphere and the Pauli matrices are thus the 14

truncate the (ˆb† + ˆb)3-term from Table III: Table III. Overview of the different combinations of the step operators and their truncation to two dimensional Pauli op- erators. Subscripts are included for the interaction terms, (ˆb† + ˆb)3 0 =(ˆb† + ˆb)2 1 and refer to different nodes. All constant terms have been | i | i  =(ˆb† + ˆb) √2 2 + 0 neglected. | i | i =√6 3 + 3 1 , Step operators Pauli operators | i | i  ˆb† ˆb iσy (ˆb† + ˆb)3 1 =(ˆb† + ˆb)2 √2 2 + 0 − − | i | i | i ˆb† + ˆb σx   =(ˆb† + ˆb) √6 3 + 3 1 (ˆb† ˆb)2 σz | i | i − − (ˆb† + ˆb)2 σz =√24 4 + 6√2 2 + 3 0 . − | i | i | i (ˆb† + ˆb)3 3σx Using the orthonormality of the states we obtain the (ˆb† + ˆb)4 6σz − representation of the operator † † x x (ˆb ˆbi)(ˆb ˆbj ) σ σ i − j − − i j † † x x (ˆb + ˆbi)(ˆb + ˆbj ) σ σ  ˆ† ˆ 3 ˆ† ˆ 3  i j i j ˆ† ˆ 3 0 (b + b) 0 0 (b + b) 1 ˆ† ˆ 3 ˆ† ˆ x x M2[(b + b) ] = h | † 3| i h | † 3| i (bi + bi) (bj + bj ) 3σi σj 1 (ˆb + ˆb) 0 1 (ˆb + ˆb) 1 ˆ† ˆ 2 ˆ† ˆ 2 z z z z h |  | i h | | i (bi + bi) (bj + bj ) σi σj 2σi 2σj 0 3 − − = = 3σx. 3 0

Truncation of the remaining terms are presented in Ta- z ble III. 0 If we consider the example circuit in Fig. 1, after we have removed non-conserving terms as in Eq. (69) and θ assume an anharmonicity large enough for truncation to a two-level system, we obtain the following Hamiltonian

y ˆ ω˜1 z ω˜2 z + − − + = σ1 σ2 + g12(σ1 σ2 + σ1 σ2 ), (75) x H − 2 − 2

± x y where we have defined σn = (σn iσn)/2. This is essen- tially two qubits which can interact∓ by swapping excita- 1 tion between them, i.e., interacting via a swap coupling. Figure 6. The Bloch sphere. Each point on the Bloch sphere corresponds to a quantum state. Rotations around the sphere corresponds to transformations of the state. 2. Three-level model

It can be desirable to truncate to the three lowest levels of the anharmonic oscillator, i.e., the three lowest states generators of rotations. Linear operators will then be of Fig. 5(d). This can, e.g., be useful if one want to represented by 2 2 matrices as study qutrit systems [95–97], or the leakage from the × qubit states to higher states [98, 99]. In this case the operators will be represented as 3 3 matrices, the matrix representation of the step operators× become  ˆ ˆ  ˆ 0 0 0 1 M2[O] = h |O| i h |O| i . (74)     1 ˆ 0 1 ˆ 1 0 0 0 0 1 0 ˆ† ˆ h |O| i h |O| i M3[b ] = 1 0 0 ,M3[b] = 0 0 √2 . (76a) 0 √2 0 0 0 0

In general we denote the n n matrix-representation of while the number operator becomes × an operator ˆ with Mn[ ˆ]. O O 0 0 0 ˆ†ˆ In order to apply this mapping to the Hamiltonian we M3[b b] = 0 1 0 , (76b) must map each operator in each term. As an example we 0 0 2 15 and powers of ˆb† + ˆb becomes levels of the two anharmonic oscillator modes. The trick is to rewrite the cosine in terms of exponentials that   1 0 √2 only contain one mode. We write the cosine in terms of ˆ† ˆ 2 M3[(b + b) ] =  0 3 0  , (76c) two complex exponentials as usual and note that each √2 0 5 exponential can be written as a product because the 0 3 0  operators of different (an)harmonic oscillators commute M [(ˆb† + ˆb)3] = 3 0 6√2 , (76d) 3   r r ! 0 6√2 0 ζ1 ˆ† ˆ ζ2 ˆ† ˆ cos (b1 + b1) (b2 + b2)  3 0 6√2 2 − 2 ˆ† ˆ 4  q q M3[(b + b) ] =  0 15 0  . (76e) 1 i ζ1 (ˆb†+ˆb ) −i ζ2 (ˆb†+ˆb ) = e 2 1 1 e 2 2 2 (78) 6√2 0 39 2 q q  −i ζ1 (ˆb†+ˆb ) i ζ2 (ˆb†+ˆb ) From Eq. (76e) it is clear to see the varying size of the + e 2 1 1 e 2 2 2 . anharmonicity, as the differences 15 3 = 12 and 39 15 = 24 between the levels changes. This− pattern continues− for higher levels and means that we can distinguish between From this expression it is clear that we need to find the ma- the levels. trix representation of the general operator exp[ik(ˆb† + ˆb)] p As we are dealing with 3 3 matrices we can no longer for some real number k = ζ/2. To find the desired ma- use the Pauli spin-1/2 matrices× as a basis for the operators. trix representation we consider the displacement operator In this case one could use the Gell-Mann matrices as a ˆ† ∗ˆ basis, however, often it is more convenient simply to leave Dˆ(ξ) = eξb −ξ b, (79) the step operators be. We are not limited by three levels, and it is possible to truncate the system to an arbitrary where ξ is complex number. This operator is unitary number of levels, thus creating a so-called qudit. and satisfies D(ξ)† = D( ξ), as well as the following commutation relation − 3. Exact truncation [Dˆ(ξ), ˆb†] = ξ∗Dˆ(ξ). (80) − In the above truncations to two and three levels the The operator creates coherent states by ’displacing’ the cosine contribution from the Josephson junction is ex- vacuum state panded to fourth order, which introduces some error and limits the qubits to operate in a given regime. However, Dˆ(ξ) 0 = ξ , (81) the behavior of the exact Hamiltonian should, however, | i | i be nearly the same with only negligible differences. It is of interest to find the truncated version of the entire where ξ is the coherent state defined by ˆb ξ = ξ ξ .A cosine-operator. This can be done numerically by writing coherent| i state can be written in terms of Fock-states| i | i as the node fluxes in terms of creation and annihilation op- 2 ∞ n erators, which would be represented by finite matrices in − |ξ| X ξ ξ = e 2 n . (82) numerical calculations, and calculating the matrix cosine | i √n! | i function of these. However, that requires much harder n=0 computations than if we would be able to write the trun- cated cosine directly in terms of creation and annihilation Clearly, we have operators. We therefore present a method for doing so ˆ† ˆ here. The method presented employs the displacement eik(b +b) = Dˆ(ik). (83) operator, which is often used in studies of optical phase space [100]. Using the above commutation relation, we can derive the Consider the standard cosine term of a Josephson junc- effect of the displacement operator on any other Fock- tion bridging two nodes with node fluxes φ1 and φ2. Writ- state. With that we can calculate its matrix elements, ten in terms of the creation and annihilation operators, as desired. For ease let us just write the commutation this becomes relation and coherent state for ξ = ik r r ! ˆ ˆ ζ1 ˆ† ˆ ζ2 ˆ† ˆ Dˆ(ik)ˆb† = (ˆb† + ik)Dˆ(ik), (84a) cos(φ1 φ2) = cos (b1 + b1) (b2 + b2) , − 2 − 2 ∞ 2 n − k X (ik) (77) ik = e 2 n . (84b) | i √n! | i where ζi is the impedance (Eq. (50)) of the two nodes n=0 respectively. We want to find the matrix elements of the cosine operator in Eq. (77) with respect to the lowest For truncation to the two lowest levels we need only 16 calculate three matrix elements higher lying level and de-exciting again, which affect the dynamics of the two-level subspace. ∞ 2 n 2 − k X (ik) − k 4. Exact four-level model 0 Dˆ(ik) 0 = e 2 0 n = e 2 , (85a) √ h | | i n=0 n! h | i ∞ Using the method of exact truncation the cosine term 2 n 2 − k X (ik) − k 1 Dˆ(ik) 0 = e 2 1 n = ike 2 , (85b) and the other usual operators appearing when recasting a √ h | | i n=0 n! h | i superconducting circuit Hamiltonian in terms of harmonic oscillator modes can also be truncated exactly to the four 1 Dˆ(ik) 1 = 1 Dˆ(ik)ˆb† 0 h | | i h | | i lowest levels. This can be useful for numerical studies = 1 (ˆb† + ik)Dˆ(ik) 0 of the higher levels effect on the two-level dynamics. It h | ∞ | i is advantageous to perform the truncation analytically 2 n − k X (ik) (85c) = e 2 ( 0 n + ik 1 n ) to avoid having to do it numerically, in particular the √ n=0 n! h | i h | i truncation of the cosine term. First we define some new 2 2 − k matrices = (1 k )e 2 . − 1 0 0 0  ˆ † ˆ ˆ 0 1 0 0 From D(ξ) = D( ξ) we have 0 D(ik) 1 = Z =   , (88a) − h | | i  −   † k2 0 0 3 0 ˆ † − 1 D(ik) 0 = ike 2 . With this we can conclude 0 0− 0 5 h | | i − 0 0 0 0 " k2 k2 # − − 0 0 0 0 h ik(ˆb†+ˆb)i e 2 ike 2 A =   , (88b) M2 e = 2 2 − k 2 − k 0 0 1 0 ike 2 (1 k )e 2 − (86) 0 0 0 3  2 2  2 k k z x − k = 1 + σ + ikσ e 2 , 0 0 0 0 − 2 2 0 0 0 0 B =   , (88c) 0 0 0 0 which is exactly what we need to truncate cosine- 0 0 0 1 operators. Consider again the standard Josephson junction term and finally Xij and Yij for i, j = 0, 1, 2, 3 with i < j, from Eq. (78). We can now perform exact truncation of whose (a, b)th entries are it to its lowest two levels using the above identity. We ( find 0, for ab = ij, ji (Xij)ab = 6 (89a)  q q 1, for ab = ij, ji 1 ζ1 ˆ† ˆ ζ2 ˆ† ˆ ˆ ˆ i 2 (b1+b1) −i 2 (b2+b2) cos(φ1 φ2) = e e  − 2 0, for ab = ij, ji  6 q q  −i ζ1 (ˆb†+ˆb ) i ζ2 (ˆb†+ˆb ) (Yij)ab = i, for ab = ij (89b) + e 2 1 1 e 2 2 2 − i, for ab = ji "     ζ1 ζ1ζ2 z ζ2 ζ1ζ2 z Together with the identity, Z, A, and B describe con- = σ1 + σ2 4 − 16 4 − 16 tributions to the energy levels. In particular Z can be # seen as the 4 4 expansion of σz, while A describes the ζ1ζ2 √ζ1ζ2 × z z x x −(ζ1+ζ2)/4 anharmonicity, and B describes a similar anharmonic en- + σ1 σ2 + σ1 σ2 e 16 2 ergy shift beyond the regular anharmonicity only relevant (87) for the third excited state and higher. In terms of the usual bosonic number operator nˆ = ˆb†ˆb = diag(0, 1, 2, 3), where we have ignored constant terms. Hence, the co- we may say that Z corresponds to nˆ, A corresponds to sine term has resulted in the usual contribution to the nˆ2 and B to nˆ3. Alternatively, we may say that A is qubit energies, as well as XX- and ZZ-couplings. The proportional to ˆb†ˆb†ˆbˆb and B to ˆb†ˆb†ˆb†ˆbˆbˆb, which shows difference, however, is that the coefficients of these terms how A does not affect levels below the second excited are more accurate as the calculation has not involved any one, while B does not matter for levels below the third Taylor expansions. In particular each coefficient has a excited. The Xij and Yij describe rotation or flipping factor of exp ( (ζ1 + ζ2)/4), which contains contributions between the ith and jth energy level, and are thus the corresponding− to the infinitely many possible virtual pro- expansion of σx and σy. In terms of these matrices, we cesses of exciting the anharmonic oscillator modes to any can write 17

ˆ† ˆ M4[b b] =Y01 + √2Y12 + √3Y23, (90a) − ˆ† ˆ 2 M4[(b b) ] = 2 + Z + √2X02 + √6X13, (90b) − − ˆ† ˆ M4[b + b] =X01 + √2X12 + √3X23, (90c) ˆ† ˆ 2 M4[(b + b) ] =2 Z + +√2X02 + √6X13, (90d) −  2 3 r 3/2 h i√ζ/2(ˆb†+ˆb)i ζ ζ ζ ζ ζ ζ ζ M4 e = 1 + Z + A B + i X01 X02 i X03 − 4 4 8 − 48 2 − √8 − √48 (90e)  2   2  r  2 3   ζ √6 ζ 3 ζ ζ −ζ/4 + i ζ X12 ζ X13 + i ζ + X23 e . − 4 − 4 − 6 2 − 2 24

C Section I B the Lagrangian of this circuit becomes

Cd 2 C ˙2 Cext  ˙ = φ + EJ cos φ + V (t) φ , (91) V(t) L 2 2 − where φ is the node flux. Expanding the last term, we obtain EJ Cext  2 ˙2 ˙ = 0 + V (t) + φ 2V (t)φ . (92) Figure 7. Circuit diagram of a single transmon-like super- L L 2 − conducting qubit capacitively coupled to a microwave drive line. The first term in the parenthesis is an irrelevant offset term, the second term is simply a change of the capaci- tance of the node, while the last term is our driving term. With the above method of truncating to four levels, we can With this in mind we rewrite find the exact anharmonicities as the coefficients of the C + Cext ˙2 ˙ standalone Z-operators. Just as there are ZZ-couplings = φ + EJ cos φ CextV (t)φ. (93) L 2 − among spins, which change energy levels depending on the state of the system, there will couplings involving Z The conjugate momentum of the node flux, φ, is then that change the anharmonicities. But just as we only look ˙ q = (C + Cext)φ CextV (t). (94) at standalone Z-operators when determining basic energy − levels, we would not include the interaction-contributions Doing the usual Legendre transformation, our Hamilto- to the anharmonicity when calculating it. These contri- nian takes the form butions will in general also be smaller, as they originate from terms involving more node fluxes and therefore more 1 1 2 Cext = q EJ cos φ + V (t)q, (95) ζ’s. If one wishes to just find the anharmonicity without H 2 C + Cext − C + Cext finding and reducing the complete four-level Hamiltonian, | {z } | {z } Hano Hext † one can simply replace the exponentials ei√ζ/2(ˆb +ˆb) with  2  where have removed an irrelevant offset term and denoted only 1 ζ + ζ A e−ζ/4, and then find the standalone 4 8 the anharmonic oscillator part of the Hamiltonian, ano − H A matrices. We do not need to include the other terms and the eternal driving part ext. We are now ready to from Eq. (90e) as they will only contribute to interactions perform the quantization andH the driving term becomes which are not interesting in this case. i Cext ˆ† ˆ ˆext = V (t)(b b). (96) H √2ζ C + Cext − G. Driving the qubit Assuming a large enough anharmonicity we can truncate the Hamiltonian into the two lowest levels Single-qubit rotations in superconducting circuits can 1 z y be achieved by capacitive microwave control. In this ˆ = ωσ + ΩV (t)σ , (97) H −2 section we go through the steps of analyzing a microwave controlled transmon-like qubit and then generalize to a d- where ω is the qubit frequency and Ω = Cext/(√2ζ(C + level qudit. To this end we consider the superconducting Cext)). qubit seen in Fig. 7, which is capacitively coupled to We now rotate into the frame rotating with the fre- a microwave source. Using the approach presented in quency of the qubit, also known as the interaction 18 frame as discussed in Section I E. In particular we use the amplitude of the pulse, V0. The latter two can be 0 = ωσz/2 for the transformation in Eq. (60). In this controlled using arbitrary wave generators. In case one frameH − the Hamiltonian becomes wishes to implement a π-pulse one must adjust these

y x parameters such that Θ(t) = π. Note that a single π-pulse ˆR = ΩV (t) (cos(ωt)σ sin(ωt)σ ) , (98) H − corresponds to implementing a not-gate which inverts which is equivalent to the external driving part of the the qubit [101–104]. A series of singe-qubit gates can I then be performed on the qubit by applying a sequence Hamiltonian in the interaction picture, i.e., R = ext. We now assume that the voltage is sinusoidal onH theH form of pulses [105].

V (t) =V0η(t) sin(ωextt + ϕ) 1. Generalization to qudit driving =V0η(t) [cos(ϕ) sin(ωextt) + sin(ϕ) cos(ωextt)] , (99) Now let us generalize this to a d-dimensional qudit. where V0 is the amplitude of the voltage, η(t) is some Quantizing and truncating the anharmonic oscillator part dimensionless envelope function, ωext is the external driv- ing frequency and ϕ is the phase of the driving. One of the Hamiltonian in Eq. (95) to d levels, the qudit usually defines the ’in-phase component I = cos(ϕ) and Hamiltonian becomes the out-of-phase component Q = sin(ϕ)[11]. Inserting d−1 X the voltage in Eq. (99) into the Hamiltonian in Eq. (98) ˆosc = ωn n n , (104) we obtain H n=0 | ih | 1 ˆ  x where ωn is the energy of qudit state n . This is a R = ΩV0η(t) (Q sin(δt) + I cos(δt)) σ | i H 2 − (100) rewriting of the ωˆb†ˆb-term and the anharmonicity term, + (Q cos(δt) I sin(δt)) σy, where the anharmonicity have been absorbed in to the − set of ωn. Now, starting from Eq. (96) and for simplicity where δ = ω ωext is the difference between the qubit fre- setting the phase in Eq. (99) to π/2, such that V (t) = quency and the− driving frequency and we have neglected V0 cos(ωextt), we can move to the rotating frame as was fast oscillating terms, i.e., terms with ω + ωext, in ac- also done above for the qubit using Eq. (60). We choose cordance with the rotating wave approximation. This the frame rotating with the external driving frequency Hamiltonian can be written very simple in matrix form d−1 1  −i(δt−ϕ) X ˆ 0 e 0 = n ωext n n , (105) R = ΩV0η(t) i(δt−ϕ) . (101) H | ih | H −2 e 0 n=0 from this we conclude that if we apply a pulse at the which is contrary to what we did for the qubit, where we rotated into a frame equal to the qubit frequency. We see qubit frequency, i.e., ωext = ω, we can rotate the state of that for a qubit (d = 2) we get 0 = ωextσz/2 up to a the qubit around the Bloch sphere, see Fig. 6, by setting H − ϕ = 0, i.e., using only the I-component we rotate about global constant, which we could have also chosen to use the x-axis, and by setting ϕ = π/2, i.e., using only the above, instead of the qubit frame. Q-component we rotate about the y-axis. Applying Eq. (60) to the qudit Hamiltonian in Eq. (104), We consider now the unitary time-evolution operator of we get: the driving Hamiltonian. At qubit frequency, i.e., δ = 0 d−1 it takes the form X ˆosc,R = (ωn nωext) n n . (106) H − | ih |  Z t  n=0 0 0 ˆ(t) = exp i ˆR(t ) dt U H The same transformation is performed on ext by using 0 (102) H  i  the standard expansion of the bosonic operators. Thus we = exp Θ(t)(Iσx + Qσy) . can find, by expanding the cosine in the voltage drive using −2 Euler’s formula, the total Hamiltonian in the rotating This is known as Rabi driving and can be used for engi- frame neering efficient single-qubit gate operations. Note that d−1 this holds only for δ = 0, as here the Hamiltonian com- X ˆR = δn n n + iΩn ( n + 1 n n n + 1 ) , mutes with itself at different times. For non-zero δ, one H n=0 | ih | | ih | − | ih | needs to solve the full Dyson’s series in principle [91]. The (107) angle of rotation is defined as where δn = ωn nωext is the detuning of the nth state Z t − 0 0 relative to the ground state driven by the external field Θ(t) = ΩV0 η(t ) dt , (103) − 0 and Cext V0 which depends on the macroscopic design parameters of Ωn = √n + 1Ω = √n + 1 (108) the circuit, via Ω, the envelope of the pulse, η(t), and C + Cext √2ζ 19 is the Rabi frequency of the kth transition. Of course we (c) Change into step operators following Table II. can also here include a phase in the drive, as discussed (d) Define frequencies, anharmonicities, and cou- above. Thus, by just using a single drive, we achieve great pling strengths for simplicity. control over this specific qudit transition. Transitions between other neighboring qudit states can be performed 9. If needed eliminate terms which only give rise to simultaneously by using a multi-mode driving field. small corrections, using time averaging dynamics as The external field thus introduces exchange terms be- presented in Section I E. This is done either by re- tween the qudit states, enabling transitions between the moving non-energy-conserving terms or by rotating two states if the effective detuning between two states to a desired frame using Eq. (60). k and k + 1, ∆k,k+1 = δk+1 δk = ωk+1 ωk ωext, is − − − 10. Assuming that the anharmonicity is sufficiently small enough (on the order of the Rabi frequency, Ωn). One thing to note is the small amount of leakage to large for each node, truncate the anharmonic oscil- other levels because of the limited anharmonicity of the lators into qudits as done in Section I F. For the transmon qubit – and thus limited detuning of unwanted case of qubits follow Table III. transitions. Luckily, this can largely be eliminated by the DRAG (Derivative Removal by Adiabatic Gate) method II. ADVANCED and its improvements [103, 106].

In this section we present some of the more advanced H. Overview of the basic method methods for analyzing superconducting electrical circuits. These methods are not essential for such an analysis, un- Here we present the an overview of the simplest ap- less mutual inductance is to be included in the analysis. As proach to analyzing superconducting circuits. Note that such they may be considered ’nice to know’. A thorough the order of some of the steps can be interchanged when understanding of the ’need to know’ methods presented analyzing specific circuits. in Section I is to be preferred before approaching this section. 1. Draw the circuit and label all components and ex- We first present some fundamental graph theory fol- ternal fluxes. lowed by a method for obtaining the Lagrangian using said graph theory, which includes mutual inductance. We 2. Label all active nodes and determine the ground then present a method for obtaining the Lagrangian of a node(s). system with dissipation. With this it is possible to find a master equation for the system, which we present together 3. Chose a spanning tree. If no external fluxes are with equations for calculating the decoherence times for present, this step can be skipped, see Section I B. two-level systems. 4. Add the energy of each element, following Table I, in order to construct the Lagrangian. A. Fundamental graph theory of electrical • The energy of components on the spanning networks tree must be added without external fluxes. • The energy of components on the closure In this section we present some fundamental definitions branches, i.e., not on the spanning tree, must from graph theory. The reason for this is that graph theory be added with external fluxes. is the natural language of electromagnetic circuits, where each circuit element can be represented as an edge on a 5. Change into Hamiltonian formalism by doing a Leg- graph. By introducing the following definitions we obtain endre transformation, see Section I B 2. a more rigorous framework for analyzing circuits. We will start by defining the fundamental concepts of graph 6. Quantize the Hamiltonian as in Section I C 1. theory. We will also describe the quantities important 7. Introduce the effective energies of the components to circuit analysis using the example circuit shown on Section I C 2. Fig. 8(a). The example circuit consists of a transmon qubit capacitively coupled to a resonator, which is a very 8. Assuming that the effective capacitive energy of each common setup [90, 107]. For more material on graph node is sufficiently larger than the Josephson energy theory see, e.g., Ref. [108]. of the node, recast the Hamiltonian into interacting harmonic oscillators, following Section I D. This is Definition 1 (Graph) A graph = ( , ) is a set G N B done following these steps: of nodes = n1, . . . , nN where N is the number of nodes, andN a set{ of branches} (sometimes called edges) = (a) Expand cosine terms from potential Josephson B b1, . . . , bB where each branch connects a pair of nodes junctions. and{ B is the} number of branches. The number of nodes is (b) Collect all second order terms. called the order of the graph and is denoted = N. We |G| 20

(a) (b)

(c)

Figure 8. (a) Example transmon-resonator circuit. The chosen spanning tree (red) consists of the resonator Lr inductance and the shunting capacitance Cs. (b) Fundamental cutsets of the circuit (in solid) with respect to the chosen spanning tree. (c) Fundamental loops of the circuit (in solid) with respect to the chosen spanning tree.

will allow multiple branches to connect the same pair of branches C = Cs,Cr,Cc , while the inductive subgraph B { } nodes. Sometimes this is called a multigraph in order to is defined by L = Lr,LJ . Note that the set of nodes distinguish it from simple graphs where only one branch are identical forB the{ capacitive} and inductive subgraph as can connect the same pair of nodes. well as the full (super)graph, i.e., C = L = . Also, even though we have assumed our graphN toN be connected,N With this definition we can consider each circuit as its subgraphs are not necessarily connected. a graph, with each element being a branch. The first The next step in the analysis is to specify a subgraph step of any circuit analysis is to label every branch of the called a spanning tree for our graph. graph. These can be labeled in different ways, usually via the element or the flux through the current. These Definition 3 (Spanning tree) A spanning tree of a are of course equivalent and often both are used as they graph is a connected subgraph that contains the same G T complement each other. nodes as (i.e., T = G) and contains no loops. G N N Using the elements as the labels, the set of branches in The branches of the spanning tree are called twigs and Fig. 8(a) becomes = Lr,Cs,Cr,Cc,LJ . The order of branches of the complement of the spanning tree are called B { } the graph is = 3 and the nodes can be labeled arbitrarily. links (or chords). Note that there are BG (NG 1) − − The numberG of branches is B = 5 and we can thus write links. the flux over all branches as a vector with five elements The spanning tree connects every pair of nodes through  T exactly one path. For our example we have chosen branch Φ = Φ1 Φ2 Φ3 Φ4 Φ5 , (109) 1 and 2 as our spanning tree as shown in red on Fig. 8. where the order of the fluxes corresponds to the order The linear inductor together with the Cs shunting capaci- of the branches in . Note that we have indicated the tor constitute the twigs of the tree while the remaining B direction of every branch in Fig. 8(a) using arrows. We capacitors (Cr,Cc) and Josephson junction (LJ ) are links. will define positive branch current Ib > 0 as the case Note that we are free to chose our spanning tree differently where current flows through a branch in the direction of as long as it obeys the definition. We could for example the arrow. Using the passive sign convention, the voltage have chosen the inductive subgraph, as mentioned above, start end over a branch is then given by Vb = V V , which however, we could not have chosen the capacitive sub- b − b ensures that the power Pb = IbVb is positive if energy is graph as it includes a loop. This freedom in choosing being stored or dissipated in the branch element. Strictly the spanning tree corresponds to a gauge freedom in the speaking, this makes our graph a directed graph, but since equations of motion. all electrical network graphs are directed graphs, we are Choosing a spanning tree also allows us to define funda- simply going to call them graphs. We are also going to mental cutsets and fundamental loops, which are impor- assume that our graph is connected, meaning that there tant when deriving the equations of motion for a circuit. exists a path between every pair of nodes. We will start with the fundamental cutsets.

Definition 2 (Subgraph) A graph = ( H, H) is Definition 4 (Cut) Given a graph = ( , ) a cut is H N B G N B called a subgraph of = ( G, G), written , if a partitioning of nodes into two disjoint sets A and G N B H ⊆ G N N H G and H G. If is a subgraph of but B. With every cut we can associate a cutset, which is N ⊆ N B ⊆ B H G N = it is called a proper subgraph. the set of branches that have endpoints in both A and H 6 G N B. In the electrical circuit setting, the notion of subgraphs N are often used to describe the capacitive and inductive Note that removing a single twig cuts the spanning tree, subgraphs of the circuit. In the case of the example in , into two disjoint subgraphs with nodes A and B. Fig. 8(a) the capacitive subgraph is defined to be the set of SuchT a cut is called a fundamental cut, and theN branchesN 21 that must be removed in order to complete the same cut In other words we iterate through the branches and the on the full graph is called a fundamental cutset. More set of f-loops. If the given branch is in the given f-loop the formally: matrix entry becomes 1, with a plus if the branch has the same orientation as± the f-loop (which is determined Definition 5 (Fundamental cut) Given a graph by the link of the f-loop). If the branch is not in the given G and a spanning tree we define a fundamental cut f-loop the matrix entry is 0. T or f-cut as a cut whose cutset contains only one twig. Consider our example circuit and its fundamental loops from Fig. 8(c). The first fundamental loops consists of In practice the fundamental cutsets can be found by the link Φ3 and the twig Φ1. The orientation of the loop removing one twig from the spanning tree. This creates (determined by Φ3) is clockwise, which means that the two disjoints subgraphs of the spanning tree with nodes (L) F11 = 1, since the twig Φ1 points in the anti-clockwise A and B. Now remove the links of the full graph with − N N direction. The only other non-zero entry in the first row endpoints in both partitions. The cutset is then the set (L) of all the removed links and the single twig. We thus end is F13 = 1, corresponding to the link Φ3 oriented in the up with a unique cutset with one twig and any number of clockwise direction. Following the same method for the links. This can be done for every twig, and the number of other two f-loops we find fundamental cutsets is thus equal to the number of twigs  1 0 1 0 0 = N 1. The fundamental cutsets of our example (L) −1 1 0 1 0 graph|T | can− be seen in Fig. 8(b). F =   . (111) 0 −1 0 0 1 We now turn our attention to the loops. By taking the − spanning tree and adding a single link from the full graph As with the loops, we can also choose an orientation we form a unique loop. Such a loop contains exactly for the cutsets. If a cutset is oriented from to , we one link and one or more twigs. We call these loops the A B say that a branch in the cutset has positiveN orientationN fundamental loops of the with respect to the spanning G if it begins in A and ends in B. We choose to orient tree . N N T every f-cutset such that its twig in an f-cutset has positive Definition 6 (Fundamental loop) Given a graph orientation. We can then define the fundamental cutset and a spanning tree we define a fundamental loopG matrix. T or f-loop as a loop consisting of exactly one link and one Definition 8 (Fundamental cut matrix) Given a or more twigs. connected graph = ( , ), with spanning tree , we define the fundamentalG N cutB matrix, or f-cut matrixT , The number of fundamental loops that can be formed F (C) as is equal to the number of links. The fundamental loops of our example graph can be seen in Fig. 8(c).  +1 if bj ci and ti, bj same orientation As we shall see in the following section the fundamental (C)  ∈ Fij = 1 if bj ci and ti, bj opposite orientation , loops and cuts allows us to write Kirchhoff’s laws in a − ∈ 0 if bj / ci compact and useful way. ∈ (112) where 1 i = N 1 and 1 j B and ti is the ≤ ≤ |T | − ≤ ≤ 1. Circuit matrices twig of the ith cutset, ci. In other words we iterate through the branches and the Using the notion of f-loops and f-cuts, we define two set of cutsets. If the given branch is in the given cutset the characteristic matrices for the network graphs. matrix entry becomes 1, with a plus if the branch has For every loop we can define the orientation, i.e., clock- the same orientation as± the cutset (which is determined wise or anti-clockwise. For an f-loop we will let the orien- by the orientation of the twig of the cutset). If the branch tation be determined by the orientation of its link. We is not in the given f-cutset the matrix entry is 0. can then define the fundamental loop matrix. As an example take the first cutset from Fig. 8(b). The Definition 7 (Fundamental loop matrix) Given a twig Φ1 and link Φ3 both point towards the same node and graph = ( , ), with spanning tree , we define the thus have positive orientation. The final link Φ4 points fundamentalG N B loop matrix, or f-loopT matrix, F (L) away from the node and has negative orientation. Thus as the first row of the cutset matrix becomes [1, 0, 1, 1, 0]. By analyzing the other cutset in the same manner,− we  +1 if bj fi and li, bj same orientation find the fundamental cutset matrix  ∈ F (L) = , ij 1 if bj fi and li, bj opposite orientation 1 0 1 1 0 − ∈ F (C) = . (113) 0 if bj / fi 0 1 0− 1 1 ∈ (110) where 1 i = B (N 1) and 1 j B and All branches of the graph are either twigs or links. ≤ ≤ |G\T | − − ≤ ≤ li is the link in the ith f-loop, fi. Every f-cutset contains only one twig, and every f-loop 22 contains only one link. Additionally, for every partition where we have sn,b = +1 if the branch b ends at node of nodes defined by an f-cut, every f-loop must begin and n and sn,b = 1 if b begins at n. This is equivalent to end in the same partition. Thus every f-cutset and f-loop the definition− in Eq. (3a). Recall that a cutset is the set share either 0 or exactly 2 branches. Now consider the of branches between two partitions of nodes. Thus if no elements charge is to accumulate at a single node, the total current from one partition of nodes to another must be zero. We  (L) (C) T  X (L) (C) F (F ) = Fik Fjk . (114) can write this using the f-cut matrix as ij k F (C)I = 0. (118) Evidently, the (i, j)th element depends only on the ith b. Kirchhoff’s voltage law states that if we choose f-loop and the jth f-cut. If the f-cutset and f-loop share no some oriented loop of branches L, the algebraic sum of branches all the terms are zero, and in the case where they voltages around the loop must equal the electromotive share exactly two branches we get two non-zero terms force induced by external magnetic flux, Φ˜ L, through the with opposite sign. We thus have face enclosed by the loop, i.e.

(L) (C) T X ˙ F (F ) = 0. (115) sL,bVb = Φ˜ L, for all loops L, (119) b∈L Multiplying Eqs. (111) and (113) we see that this is exactly where sL,b = +1 if b is oriented along L, and sL,b = 1 the case for the example graph, as it should be. if b is oriented against L. The external flux through the− loop L is denotes Φ˜ L. This is equivalent to the definition in Eq. (3b). Thus, the f-loops of the graph define a set of B. Method of electrical network graph theory equations and using Eq. (2a) we may write Kirchhoff’s voltage law as In this section we present a more mathematical method F (L)Φ = Φ˜, (120) for obtaining the Hamiltonian of an electrical supercon- T ducting circuit. This method is based upon Ref. [58] and where Φ˜ = (Φ˜ 1,..., Φ˜ B−N+1) is the vector external uses electrical network graph theory [65]. This method fluxes through the fundamental loops. can be advantageous over the method presented in Sec- c. Reducing the number of coordinates Using Kirch- tion I B for larger and more complex circuits. hoff’s voltage law, we are able to reduce the number of The first step is to label and order all the circuit el- free coordinates. In fact we only need to specify the fluxes ements (branches) of the network graph, and choose a of the spanning tree in order to calculate the remaining spanning tree for the graph. Without loss of generality, fluxes. In order to do so, we will write our f-cut matrix as we will order the elements such that the first branches F (C) = 1 F  , (121) are the twigs and we can write the fluxes and|T | currents over all elements as vectors where F is a = (N 1) (B N + 1) matrix and the identity|T | × is |G\T a (N | 1) − (N× 1)− matrix. Note Φ  I  − × − Φ = t , I = t , (116) that our specific ordering of the circuit elements (twigs Φl Il first, then links) allows for the simple block structure of Eq. (121). This structure is clearly seen in the example where Φt (It) are the fluxes (currents) of all the twigs and in Eq. (113). Reordering the elements simply shuffles the Φl (Il) are the fluxes (currents) of all the links. After all rows and columns of the fundamental cut matrix, and elements have been labeled and a tree has been selected, the following derivations can easily be generalized. From (L) we construct the fundamental matrices of the graph F Eq. (115) and Definition 7 we find that we can write the (C) and F following Definitions 7 and 8 respectively. In f-loop matrix in a similar manner the following we show how these matrices may be used F (L) =  F T 1 , (122) to set up the equations of motion and reduce the number − of free coordinates. where F is the same matrix as in Eq. (121), meaning that the identity is now (B N + 1) (B N + 1). This structure is again seen in− the example× in− Eq. (111). We 1. Kirchhoff’s laws can then rewrite Kirchhoff’s voltage law in Eq. (120) and isolate the fluxes of the links Using Eq. (116) and the f-matrices, we will reformulate T Φl = Φ˜ + F Φt, (123) Kirchhoff’s laws as stated in Eq. (3). a. Kirchhoff’s current law states that no charge may and use this to write our flux vector in Eq. (116) in terms accumulate at a node. Mathematically we may write this of the twig and external fluxes as     Φt (C) T 0 Φ = T ˜ = (F ) Φt + ˜ , (124) X F Φt + Φ Φ sn,bIb = 0, for every node n, (117) b incident on n meaning that we have eliminated the fluxes of the links. 23

2. Equations of motion (without dissipation) which means that L0 must be positive semi-definite. We will further assume L0 is positive definite meaning that 0 < I0T L0I0 for I0 = 0. This assumption is also phys- In this section we will use Kirchhoff’s current law to L L L 6 set up the equations of motion for the system. For this ically sensible since any current through the inductors purpose it is convenient to introduce the species-specific must store at least some magnetic field energy in a realis- tic configuration. It also ensures that the symmetric L0 vectors IS and ΦS ( matrix is invertible and we can write Ii if the ith element is of species S, 0 0−1 0 (IS)i = , I = L Φ . (130) 0 otherwise, L L (125a) We can expand the matrix L0−1 to work on the full flux ( vector by inserting zeros on the non-inductor columns Φi if the ith element is of species S, and rows. Similarly, we also build the corresponding full (ΦS)i = 0 otherwise, inductor current vector IL. The resulting equation can (125b) be written + where the species subscript, S, indicates the element IL = L Φ, (131) species, i.e., capacitor, inductor, etc. This can be under- + −1 stood as the current and flux vectors with everything but where L is the matrix found by expanding L with the zero-columns and rows of the non-inductor elements. S species removed. We use C for capacitors, L for linear + inductors and J for Josephson junctions. Formally, L is the Moore-Penrose pseudo-inverse [109] The first step is to express the current of every branch in of the original full inductance matrix L. Now we only need to include the current through the terms of the tree fluxes Φt. The current flowing through a capacitor with capacitance C is given by I = CV˙ = CΦ¨ Josephson junctions, which follows from the Josephson and we can thus write the current flowing through all relation capacitors as IJ = DJ sinΦ, (132)

IC = DC Φ¨ , (126) where DJ is a diagonal matrix with the Josephson critical where DC is a diagonal matrix with the circuit capaci- currents on the diagonal, see Eq. (11) for the case of a tances on the diagonal. In this context all other circuit single Josephson junction. As with L and C, all other ele- elements are counted as having zero capacitance. ments than Josephson junction are counted as having zero T The flux stored in the linear inductors is related to the critical current. The vector sinΦ = (sin Φ1,..., sin ΦB) currents through is understood as the vector of sines of the branch fluxes. Thus, the current through each branch can be written LI = ΦL, (127) as a function of the branch flux and its derivatives as seen where L is a symmetric matrix with diagonal elements in Eqs. (126), (131), and (132), and Kirchhoff’s current law thus gives a set of coupled second order differential Lii = Li where Li is the inductance of the ith element. equations For all other elements than linear inductors we set Li = 0. The off-diagonal elements are the mutual inductances 0 =F (C)I = F (C) [I + I + I ] L = M = k pL L between the ith and jth inductor, C L J ij ij ij i j ¨ ˙ with 1 < kij < 1 being the coupling coefficient. If a =MΦt + Q0 + KΦt + I0 − (133) positive current in the one inductor results in a positive  0 +F (C)D sin (F (C))T Φ + , magnetic flux contribution through another we have kij > J t Φ˜ 0. If the contribution is negative, we instead have kij < 0. The numerical value of kij depends on the placement of where we have defined the ’mass’ and ’spring constant’ the inductors relative to each other. matrices (analogous to in Section I B 3) Note that all the rows and columns belonging to ele- (C) (C) T ments not on the inductor subgraph are zero. By removing M =F DC (F ) (134a) these zero rows and columns we get a NL NL matrix K =F (C)L+(F (C))T , (134b) 0 × L , where NL is the number of inductors. We can then rewrite Eq. (127) as and the offset charges and flux induced currents L0I0 = Φ0 , (128) " # L L (C) 0 Q0 =F DC ˙ (135a) 0 0 Φ˜ where IL and ΦL are the corresponding vectors found by removing all the non-inductor entries of the full size 0 I =F (C)L+ . (135b) vectors I and Φ. The magnetic field energy stored in the 0 Φ˜ inductors is Note that these matrices are different from the capacitive 1 0T 0 0 0 EL = I L I , (129) ≤ 2 L L and inductive matrices presented in Section I B. 24

3. Voltage and current sources This Hamiltonian can easily be quantized using the ap- proach presented in Section I C 1, where this time the Until now we have assumed that external fluxes are canonical variables are the branch fluxes Φb and Qb of our only control parameters, but we can also add current the twigs, with the commutator relation in Eq. (42). and voltage sources. A voltage sources can be added in series with existing elements without introducing new constraints on the branch fluxes. This effectively just C. Dissipation transforms the external flux vector Z t In this section we introduce dissipation to the circuits. ˜ ˜ 0 0 Φ(t) Φ(t) VV (t ) dt , (136) We continue from the dissipation-free Hamiltonian derived → − −∞ in Section II B 2, however, dissipation can also be added where (VV )i is the voltage generated by the source on the to the formalism used in Section I B. ith branch, or 0 if the ith branch is not a voltage source, Dissipative elements, like resistors, behave irreversibly. i.e., defined analogously to Eq. (125). This is a problem as we wish to formulate a Hamiltonian Similarly, we can add a current source in parallel with formalism of the circuits, and Hamilton’s equations of mo- an existing element without introducing additional con- tion [Eq. (31)] are invariant upon time reversal. However, straints on the free currents. This simply modifies I0 it is possible to extend the Hamiltonian formalism in order according to to fix the reversability problem. On way to fix this prob- lem is called the Caldeira-Leggett model [66] and applies (C) I0 I0 + F IB, (137) to systems with linear dissipation. The Caldeira-Leggett → model is a general semi-empirical model which models where IB is the bias current vector with zeros on all the dynamics of a system coupled to an environment. In entries except those belonging to a branch with a current this model a bath (reservoir) of harmonic oscillators is source, where instead it has the applied current, i.e., as introduced in order to describe the environment, which in Eq. (125a). in our electromagnetic circuit setting is the degrees of freedom of the two-terminal linear dissipative element. 4. Lagrangian and Hamiltonian

One can show, using Eq. (27), that a Lagrangian fulfill- 1. Dissipative elements ing the equations of motion in Eq. (133) is A linear dissipative two-terminal element can be char- 1 T acterized by a frequency dependent admittance Y (ω). = Φ˙ MΦ˙ t + Q0 Φ˙ t L 2 t · Admittance is a measure of how easily a device will al- 1 T low a current to flow. It is defined as the reciprocal of Φ KΦt I0 Φt (138) − 2 t − · the impedance Z(ω) = 1/Y (ω) analogous to the rela-    (C) T 0 tion between conductance and resistance. The electrical + JC cos (F ) Φt + , · Φ˜ impedance of a circuit is defined as where we have defined the critical current vector Z = R + jX, (142)

(JC )i =(DJ )ii. (139) where R is the resistance of the circuit and the imaginary part X is the reactance. We have used the symbol j = i The conjugate momenta of the twig branches are then − given by which is standard in electrical engineering. This means that we can treat shunt resistors in a circuit as external ∂ impedances. Q = = MΦ˙ + Q , (140) t L t 0 The idea of the Caldeira-Leggett model, in the context ∂Φ˙ t of electrical circuits, is to replace the linear dissipative and the Hamiltonian can be found using the Legendre element, characterized by admittance Y (ω), by an infinite transformation series of LC oscillators in parallel, see Fig. 9. The internal degrees of freedom of the admittance can be seen as =qt Φ˙ t H · − L the fluxes of the intermediate node in the LC oscillators. 1 T −1 This series of harmonic oscillators constitutes the bath = (Qt Q0) M (Qt Q0) 2 − − and it is indeed the passage from a finite number of 1 T (141) degrees of freedom to an infinite number that reconciles + Φ KΦt + I0 Φt 2 t · the irreversible behavior on physical time scales and the    (C) T 0 formal reversibility of Hamilton’s equations of motion. JC cos (F ) Φt + . − · Φ˜ The admittance of each oscillator k in the series is given 25

the Fourier transform one must replace ω ω + i with →  0+. L L L → Y(ω) 1 2 k In the frequency domain the voltage of the external impedances in a circuit can be described as 1 2 k C1 C2 Ck VZ (ω) = Z(ω)IZ (ω), (146)

Figure 9. Caldeira-Leggett model of an admittance Y (ω). where IZ (ω) is defined as in Eq. (125) with Z indicating The two-terminal dissipative element can be represented as impedances and VZ (ω) is defined similarly, while Z(ω) an infinite number series of LC oscillators in parallel. Each is the impedance matrix with the external impedances inductor in the series has inductance Lk, while the capacitors on the diagonal and zero on the diagonal elements corre- has capacitance Ck. In between is a node φk. sponding to branches with no external impedances. The external impedances can also be described in the time domain by

VZ (t) = (Z IZ )(t), (147)  1 −1 ∗ Yk(ω) = jLkω + , (143) jCkω where the asterisk denotes convolution and is purely imaginary. When we take the infinite limit Z t we obtain both an imaginary and a real part of the ad- (f g)(t) = f(t τ)g(τ) dτ . (148) ∗ −∞ − mittance, with the real part being responsible for the resistance of the circuit. Using the admittance we can also express the current of Following the Caldeira-Leggett model we then replace the impedances as a function of their branch fluxes by the real part of the admittance function Y (ω) with an rewriting Eq. (146) infinitely dense comb of δ functions. The relevant quantity for a good description of the dissipation mechanism is the IZ (ω) = Y (ω)Φ˙ Z (ω), (149) bath spectral function where Y is the matrix of admittances, corresponding to ∞ 2 −1 π X κk Z but only taking the inverse of the non-zero diagonal J(ω) = δ(ω ωk), (144) 2 mkωk − elements. Formally this is equivalent to taking the Moore- k=1 Penrose pseudo-inverse as done with the inductive matrix in Eq. (131), however, much more simple as there are no where mk and ωk are the analogous mass and frequency of non-diagonal elements. Changing into the time domain the kth bath harmonic oscillator, while κk is the coupling parameter between the system and the kth harmonic we find oscillator. The spectral bath function puts a constraint IZ (t) = (Y iωΦZ )(t) (150) on the choice of κk depending on the desired form of J. ∗ Consider the form J ωα. When α = 1 the dissipation ˙ corresponds to the classical∝ ohmic dissipation. When where the factor comes from the fact that ΦZ (ω) = α > 1 the dissipation is called super-ohmic, and when iωΦZ (ω), which can be seen by Fourier transforming ˙ α < 1 it is called sub-ohmic. ΦZ (ω) and performing integration by parts. In order to add external impedances to the model we must thus add (C) a term F IZ to Eq. (133) such that it becomes 2. Adding impedances to the equations of motion (C) (C) 0 =F I = F [IC + IL + IZ + IJ ]

Both the admittance and impedance can be defined in =MΦ¨ t Q˙ 0 + KΦt + I0 + Md Φt − ∗ (151) the time domain instead of the frequency domain. These  0 two domains are related by a Fourier transformation +F (C)D sin (F (C))T Φ + , J t Φ˜ Z ∞ Z ∞ Y (ω) = Y (t)eiωt dt = Y (t)eiωt dt , (145a) where we have defined the dissipation matrix −∞ 0 Z ∞ Z ∞ (C) (C) T Z(ω) = Z(t)eiωt dt = Z(t)eiωt dt , (145b) Md(ω) = iωF Y (F ) , (152) −∞ 0 and we have assumed no external flux on the impedance where we use the notation that f(ω) is the Fourier trans- branches. This matrix is in fact real and symmetric form of f(t). We have also used that both Y (t) and Z(t) since the admittance matrix, Y , is purely imaginary and are causal (or one-sided) functions meaning that they diagonal, which means that we can apply the Caldeira- are zero for t < 0. In order to ensure convergence of Leggett system bath model [66]. 26

3. Single impedance dynamics of the bath is given by

∞  2  In the lowest-order perturbation theory of the equa- ˆ 1 X pˆk 2 2 B = + mkωkϕˆk , (155) −1 H 2 mk tions of motion in the dissipative parameter Yi = Zi k=1 (which we assume to be small), the contributions to Md from different external impedances are additive. This where we have redefined mk = Ck as the mass of the kth fictitious harmonic oscillator and ω = 1/√L C as the means that one can calculate Md for each impedance Zi k k k separately, while the inverse of the remaining impedances frequency of the kth harmonic oscillator. As this term −1 only describes the dynamics of the external impedance vanishes, Zj6=i 0, and then add the contributions to obtain the full→ coupling Hamiltonian. Similarly, deco- it is often of little interest. Much more interesting is the herence rates due to different impedances are additive in coupling term between the bath and the system, lowest-order perturbation theory. If Y can be written as a ∞ ∞ 2 X 2 X κ sum where each term contains only one impedance Zi the ˆ ˆ ˆ k SB = m Φ κkϕˆk + (m Φ) 2 , (156) H · · 2mkω contributions to Md from different external impedances k=1 k=1 k are additive in the exact case. Due to the above we will only consider the case of a where we have redefined the coupling constant κk = 2 0 mkω κ . The first term in SB is the actual coupling, single impedance, as we can use lowest-order perturbation − k k H theory to describe the dynamics of a system of several while the second term is a counter term that compensates external impedances, simply by adding the impedances the energy renormalization caused by the first term. This and modelling them as a single external impedance. term makes sure that the dissipation is homogeneous over In the case of a single external impedance, only a single all space, but it does not play a role in the two-level entry is non-zero in Y (ω). This means that we can define approximation for qubits. the scalar function K(ω) as iω times this non-zero entry We now turn to the equations of motion of the full Hamiltonian, i.e., ˆ = ˆB + ˆS + ˆSB. Using Eq. (31) in Y (ω). The single non-zero entry in Y (ω) picks out a H H H H single column of the F (C) matrix in Eq. (152) and defining the equations of motion for the bath variables is this column as m¯ , we can further define the normalized ˙ ¨ 2 pˆk = mkϕˆk = mkω ϕˆk κkm Φˆ. (157) vector m = m¯ /√µ parallel to m¯ , where µ = m¯ 2. We − k − · | | can then write the dissipation matrix as Changing into the frequency domain we find that ϕ¨k = 2 T ω ϕk, which we use to solve for the bath variables Md(ω) = µK(ω)mm . (153) − m Φˆ T Note that µ is the eigenvalue of the matrix m¯ m¯ . ϕˆk = κk 2· 2 (158) mk(ω ω ) − k Similarly we find the equations of motion for the flux 4. The Caldeira-Leggett model variables of the system

∞ ∞ 2 As mentioned in Section II C 1 the Caldeira-Leggett 2 ˆ 0 ˆ X ˆ X κk ω CΦ = U (Φ) m κkϕˆk m(m Φ) 2 . model models dissipative elements as an infinite sum of − − − − · mkω k=1 k=1 k LC oscillators. A single impedance modelled as an infinite (159) series of LC oscillators, see Fig. 9, has the Hamiltonian Inserting Eq. (158) into the above equation and rearrang- ing, we obtain ∞ " 2 0 ˆ 2 # ˆ 1 X pˆk (ϕ ˆk κkm Φ) Z = + − · , (154) ∞ 2 2 H 2 Ck Lk 2 0 X κ ω k=1 ˆ ˆ ˆ k ω CΦ = U (Φ) m(m Φ) 2 2 2 , − − − · mkω (ω ω ) k=1 k − k where Ck and Lk are the capacitance and inductance of (160) the kth harmonic oscillators respectively and ϕˆk and pˆk where we have used the identity are the (fictitious) flux variable and conjugate momentum of the kth harmonic oscillator respectively, obeying the 1 1  ω2  = 1 , (161) usual commutator relation in Eq. (40). Finally m Φˆ rep- 2 2 2 2 2 ω ωk ωk ω ωk − resents the branch fluxes which couples to the impedance.· − − Here m picks out the branch in the vector of fluxes Φˆ to show that the last term of Eq. (159) compensates part which couples to the impedance via the coupling parame- of the coupling term. 0 ters κk, which depend on the details of the coupling. The Comparing Eq. (160) to the Fourier transform of Eq. (151) and using the single impedance assumption ˆZ term must be added to the dissipation-free Hamilto- nianH in Eq. (141). in Eq. (153) we find that

Now we rewrite this, isolating the term describing the 2 ∞ 2 dynamics of the bath and the term describing the inter- ω X κk K(ω) = 2 2 2 . (162) action between the bath and the rest of the system. The µ mkω (ω ω ) k=1 k − k 27

Using the spectral function defined in Eq. (144) we can • Tr ρ2 Tr ρ, with the equality if and only if ρ = rewrite this as ψ ψ ≤for some unit state vector ψ . | ih | | i ω2 2 Z ∞ dω0 J(ω0) • Eigenvalues are larger than or equal to zero, K(ω) = . (163) µ π ω0 ω2 ω02 λ ρ λ 0, when λ is an eigenstate of ρ 0 − h | | i ≥ | i ˆ Due to the fact that K(ω) is a function of the exter- • The expectation value of an operator can be found by ˆ = ψ ˆ ψ = Tr( ˆρ), where Oρ = ψ ψ nal impedance Z(ω) and the fact that we have already hOi h |O| i O | ih | made the replacement ω ω + i in order to ensure the → An important point of the density matrix formalism is that convergence of Eq. (145), we can now apply the Sokhotski- it includes off-diagonal elements. Consider e.g., + = Plemelj theorem from complex function theory. Doing so ( 0 + 1 )/√2. In the computational basis its density| i we find matrix| i | becomesi 2  Z ∞ 0 0 0  ω 2 dω ω J(ω ) 1 K(ω) = P.V. 0 2 02 iJ(ω) , (164) ρ = + + = ( 0 0 + 1 1 + 1 0 + 0 1 ). (167) µ π 0 ω ω ω − 2 − | ih | | ih | | ih | | ih | | ih | where P.V. indicates the principal part (or principal value) This should be compared to a statistical mixture P of the integral. If we compare K(ω) to the spectral pi φi φi , for an ortonormal basis φi , which has i | ih | {| } function in Eq. (144) we see that the imaginary part no off-diagonal elements. of K(ω) is equal to the spectral function up to some Just as the Schr¨odingerequation describes the time factor, J(ω) = µ Im K(ω) . We can also relate the real evolution of the pure states of a quantum system, we and imaginary− part of{ K(ω}) using the Kramers-Kronig can find an equation for the time evolution of the density relation matrix of the system. This equation is the Liouville-von Neumann equation and is given as 2 Z ∞ dω0 ω0 Im K(ω0) Re K(ω) = P.V. { }. (165) ˆ ˆ { } −π ω0 ω2 ω02 ρ˙(t) = i[ , ρ(t)] = ρ(t), (168) 0 − − H L This means that if we experimentally measure the spectral where ˆ is the Hamiltonian of the system. Note how H function of some external impedance we can determine the first equality resembles Heisenberg’s equations of mo- both the real and imaginary part of K(ω). tion in Eq. (61), but with a different sign due to the fact that the density matrix is a dynamical variable. In the second equality we have used the Liouville operator, 5. Master equation which is defined through the condition that ˆρ is equal to i times the commutator between the HamiltonianL of − In the above section we found the equations of motion the system and the density matrix. The Liouville opera- describing the dynamics of inductors, capacitors, Joseph- tor is sometimes called a superoperator due to the fact son junctions and the dissipative elements. Notwithstand- that it operates on an operator yielding another operator. ing, we are not interested in the dynamics of the dissipa- Writing the von Neumann equation using the Liouville tive elements, only in their effect on the dynamics of the operator makes it analogous to the classical Liouville equa- circuit elements. We therefore derive a master equation tion, where the Liouville operator is defined through the for the dynamics of the circuit variables only. A master Poisson bracket, see Section I C 1. Analogous to how the equation describes the time evolution of a system (in our exponential of the Hamiltonian governs the time evolution case an electrical circuit) where we model the system as of states it is possible to show that the Liouville operator a probabilistic combination of states at any given time, governs the time evolution of density matrices and where we can determine the transition between the Lˆt −iHˆt iHˆt states by a transition rate matrix. [110] ρ(t) = e ρ(0) = e ρ(0)e . (169) However, before finding such a master equation we Now consider the Hamiltonian of the system and the briefly introduce the density matrix formalism. Consider bath, ˆ = ˆS + ˆB + ˆSB as derived in Section II C 4. a quantum system described by a set of states ψi . We H H H H {| i} In this case the von Neumann equation, Eq. (168), de- define the density matrix of the system as scribes the dynamics of the whole system, i.e., both the X superconducting phases and the bath modes of the exter- ρ = wi ψi ψi , (166) | ih | nal impedances. In order to simplify the calculations we i change to the interaction picture, see Eq. (61), meaning P that we only need to consider the interaction part of the where wi are weight satisfying i wi = 1. The density matrix has the following properties: Hamiltonian, i.e., SB. When we take the integral form of Eq. (168) in theH interaction picture we obtain Hermitian, ρ† = ρ • Z t 0 0 0 ρ(t) = ρ(0) i dt [ ˆSB(t ), ρ(t )], (170) • Normalized, Tr ρ = 1 − 0 H 28 where the interaction Hamiltonian in the interaction pic- still depends upon an explicit choice for the initial prepa- ture is defined as ration at time t = 0. In order to fix this we substitute t0 for t t0 in the integral and extend the upper limit of ˆ ˆ ˆ ˆ ˆ i(HS +HB )t ˆ −i(HS +HB )t − SB(t) = e SBe . (171) the integral to infinity. This is allowed if the integrand H H disappears sufficiently fast for t0 much larger than the As we are not interested in the dynamics of the dissipative timescale of the bath. This is equivalent to assuming elements, we take the partial trace over the bath modes, that the time scale of the bath is much shorter than the which yields the density matrix of the superconducting smallest time scale of the system. This can be interpreted phases only as the dynamics of the system not being correlated with the dynamics of the system in the past. This typically ρS = TrB ρ(t). (172) leads to exponential decay in coherence and population. With the Markovian approximation we find Inserting Eq. (170) into Eq. (168) and taking the partial trace we find Z ∞ 0 ˆ ˆ 0 Z t ρ˙S(t) = dt TrB[ SB(t), [ SB(t t ), ρS(t) ρB]]. ρ˙ (t) = dt0 Tr [ ˆ (t), [ ˆ (t0), ρ(t0)]], (173) − 0 H H − ⊗ S B SB SB (177) − 0 H H This equation is called the Markovian master equation or where we have assumed that TrB[ ˆSB(t), ρ(0)] = 0, mean- Redfield equation [111], and the combined above approxi- ing that the interaction between theH system and the bath mations are usually termed the Born-Markov approxima- does not influence the initial state of the whole system. tion. Note that the Markov approximation is not always Equation (173) can in principle be solved, which yields appropriate [112, 113]. an exact result describing the time evolution of the system Consider now the eigenbasis n of the system not {| i} ρS. However it is difficult to calculate, as it involves including the bath, i.e., we have ˆS n = ωn n . By the diagonalization of the complete problem. Therefore, taking the matrix elements of Eq. (177)H | i we obtain| i in order to simplify the problem, we assume the weak coupling limit, i.e., the limit where the coupling between X ρ˙nm(t) = Rnmklρkl(t), (178) the system and the bath is small. Weak coupling is − satisfied if kl

J(ωij) J(ω) kBT where ρnm = n ρ m and we have defined the Redfield 1 and 1, (174) tensor h | | i ωij  ω ω→0 ωij  Z ∞ holds for all transition energies ωij = ωi ωj of the system. ˆ ˆ − Rnmkl = dt TrB n [ SB(t), [ SB(0), k(t) l(t) ρB]] m , Here J(ω) is the spectral function in Eq. (144), kB is 0 h | H H | ih | | i the Boltzmann constant and T is the temperature. The (179) requirements in Eq. (174) can be satisfied even for a strong where we have inserted the completeness relation and the coupling to the external impedances, however in that case time dependent states of the system are given as we must require that the impedances and resistances are large compared to the quantum of resistance iHˆ t iω t k(t) = e S k = e k k . (180) | i | i | i e2 Zi,Ri . (175) If we evaluate the commutators in the Redfield tensor we  h find In the weak coupling regime we can justify that the bath X X modes do not change significantly, meaning that ρB is + − + − Rnmkl = δlm Γnrrk + δnk Γlrrm Γlmnk Γlmnk, time-independent, and that the full system remains sepa- r r − − rable as ρ(t) = ρS(t) ρB. This is sometimes known as (181) the Born approximation,⊗ which yields where we have defined

Z t Z ∞ 0 ˆ ˆ 0 0 + −iωnkt ρ˙S(t) = dt TrB[ SB(t), [ SB(t ), ρS(t ) ρB]]. Γlmnk = dt e TrB SB(t)lm SB(0)nkρB, − 0 H H ⊗ 0 H H (176) (182a) In order to further simplify this we make a Markov ap- Z ∞ 0 − −iωlmt proximation. First the integrand ρS(t ) is replaced with Γlmnk = dt e TrB SB(0)lm SB(t)nkρB, 0 H H ρS(t), meaning that the time development in the equa- tions of motion for the reduced system density matrix now (182b) only depends on the present state ρS(t). However, this replacement is not enough to create a master equation, where ωnm = ωn ωm and SB(t)nm = − H iHˆB t −iHˆB t since the time evolution of the reduced density matrix n e ˆSBe m . Inserting the definition h | H | i 29 of the interaction Hamiltonian in Eq. (156) we obtain is often called the secular approximation in this setting. This means that we only retain rates with n m = k l, + ˆ ˆ − − Re(Γlmnk) = l m Φ m n m Φ k (183) which means that the first two entries in p0 vanishes leav- h | · | i h | · | i R R T −βωnk/2 ing only p0 = (0, 0,R R ) while the relaxation e 1111 − 0000 J( ωnk ) , matrix becomes × | | sinh(β ωnk /2) | | π  R I  Im(Γ+ ) = l m Φˆ m n m Φˆ k (184) R0101 R0101 0 lmnk I R h | · | i h | · | i 2 R =  R0101 R0101 0  . (189) Z ∞ J(ω)  βω  − R R 0 0 R0000 + R1111 P.V. dω 2 2 ω ωnk coth , × 0 ω ωnk − 2 − The off-diagonal terms can be considered a correction where J(ω) is the spectral function from Eq. (144). to the qubit frequency, yielding the renormalized qubit I frequency ω˜01 = ω01 R0101. This leaves the relaxation matrix − 6. Two level approximation  −1  T2 0 0 ˜ −1 The main objective of this tutorial is to create a de- R =  0 T2 0  (190) −1 scription of two-level systems in superconducting circuits, 0 0 T1 i.e., superconducting qubits. We therefore prepare the system in the two lowest energy states, 0 and 1 , and where we have denoted the diagonal elements as the in- | i | i verse of the longitudinal decoherence time T and the assume all rates Rnmkl for n, m = 0, 1 and k, l = 0, 1 are 1 6 transversal decoherence time T , which are given as negligible compared to rates Rnmkl for n, m, k, l = 0, 1. 2 In other words this means that we neglect all transitions 1 + +  other than those between the two lowest states. This as- =2 Re Γ0110 + Γ1001 , (191a) T1 sumption can be justified at low temperature, βω12 1.  1 1 1 With this assumption we can restrict our description of = + , (191b) the system to the two lowest levels. This is equivalent T2 2T1 Tφ to the truncation done in Section I F 1. In this case the 1 + + +  = Re Γ1111 + Γ0000 2Γ0011 , (191c) density matrix becomes a 2 2 matrix which can be Tφ − described by the Bloch vector× where we have defined the pure dephasing time Tφ. The  ρ + ρ  01 10 longitudinal decoherence time, T1, describes depolariza- p = Tr(σρ) = i(ρ01 ρ10) , (185) − tion along the qubit quantization axis, often referred to ρ00 ρ11 − as “energy decay” or “energy relaxation”, which is why it is often referred to as the relaxation time. Longitudinal T where σ = (σx, σy, σz) is the vector of Pauli matrices. relaxation is caused by transverse noise, via the x- or Using the Bloch vector to write the Redfield equation in y-axis on the Bloch sphere, see Fig. 6. Depolarization Eq. (178) we obtain the Bloch equation (in the interaction of the superconducting circuit occurs due to exchange of picture) energy with the environment, leading both to excitation and relaxation of the qubits, meaning that one can write p˙ = Rp + p0, (186) − 1 1 1 with the relaxation matrix = + . (192) T1 T+ T−  R R I I R R  R0101 + R0110 R0101 R0110 R0100 R0111 I I R − R I − I Due to Boltzmann statistics and the fact that super- R =  R0101 R0110 R0101 R0110 R0100 + R0111 , − R− R− − R R conducting qubits are operated at low temperatures 2R0001 R0001 R0000 + R1111 (187) (T . 20 mK) and with a frequency in the GHz regime, the and qubits generally lose energy to the environment, meaning that the excitation rate 1/T+ is suppressed exponentially.  R R  (R0111 + R0100) The pure dephasing time Tφ describes depolarization in − I I p0 =  R0100 + R0111  , (188) the x-y plane of the Bloch sphere. It is referred to as “pure R R (R0000 R1111) dephasing,” to distinguish it from other phase breaking − − processes such as energy excitation or relaxation. Pure R I where Rnmkl is the real part of Rnmkl and Rnmkl is the dephasing is caused by longitudinal noise that couples to imaginary part. In the above we have used facts such the qubit via the z-axis. This longitudinal noise causes + − ∗ ∗ that Γnklm = (Γmlkn) and Rnmkl = Rmnlk. the qubit frequency, ω01, to fluctuate such that it is no If the qubit frequency is much larger than the rates, i.e., longer equal to the interaction frame frequency, causing ω01 Rnmkl, we can apply the rotating wave approxi- the Bloch vector, p, to precess forward or backward in mation as discussed in Section I E. This approximation the interacting frame. 30

The transverse relaxation time T2 describes the loss 105 of coherence of a superposition state pointed along the x-axis on the equator of the Bloch sphere. It is caused Phase qubit both by longitudinal noise, which fluctuates the qubit 104 frequency and leads to pure dephasing, and by transversal noise, which leads to energy relaxation of the excited-state 3 component of the superposition state. The transverse 10 relaxation rate is therefore given as the sum of half the C longitudinal decoherence rate and the pure dephasing 2 /E 10 0-π (θ-mode) J Transmon

rate, see Eq. (191b). For more on decoherence see Ref. E [11]. (C-shunted) 1 Flux qubit Using Eq. (183) we find expressions for the longitudinal 10 0-π (ϕ-mode) decoherence and the pure dephasing Fluxonium   0 1 2 βω01 10 =4 0 m Φˆ 1 J(ω01) coth , (193a) Quasicharge qubit T1 | h | · | i | 2 Cooper pair box

1 J(ω) 1 ˆ ˆ 2 10− =2kBT 0 m Φ 0 1 m Φ 1 , 2 1 0 1 2 3 T ω 10− 10− 10 10 10 10 φ | h | · | i − h | · | i | ω→0 E /E (193b) L C from which the transversal decoherence can be found Figure 10. Parameter space of the ’qubit zoo’. The qubits using Eq. (191b). Note that the transversal decoherence are plotted according to their effective Josephson energy, EJ , time can be twice the longitudinal decoherence time if and inductive energy, EL, both normalized by their effective the pure dephasing rate disappears. From Eq. (193b) we capacitive energy, EC . The marker indicates the type of qubits, see that the pure dephasing rate vanishes, in our theory, with yellow squares indicating phase qubits, red dots indicating when 0 m Φˆ 0 = 1 m Φˆ 1 , which can be obtained charge qubits, green triangles indicating flux qubits, and a h | · | i h | · | i blue star for the quasicharge qubit. Note that the placement by tuning the external fluxes. However, in reality the of the qubits are only approximate as the effective energies pure dephasing will never disappear entirely due to effects are not definitive. Note that the 0-π qubit is plotted twice, beyond our theory, such as higher-order corrections, other once for each of its modes, where the ϕ-mode works similar noise sources, or non-Markovian effects. to a fluxonium qubit, while the θ-mode works similarly to the transmon qubit.

III. EXAMPLES much less than the effective energy of the Josephson junc- In this section we present some examples of analysis tion. In this regime the Josephson tunneling dominates of more or less well-known superconducting qubits. We over the charging of the capacitor, which makes the an- start from some simple early single qubit designs, over the harmonicity effects very small for the lowest lying states. transmon qubit, couplings between qubits, to flux qubits. As the capacitor is analogous to the kinetic energy and In Fig. 10 we present an overview of the qubits mentioned the Josephson junction to the potential energy, the phase in the following example. regime corresponds to low kinetic energy compared to There are four fundamental types of qubits: Phase potential energy, which means that the lowest eigenstates qubits, charge qubits, flux qubits, and quasicharge qubit. are close to the bottom of the potential. A consequence These qubits can be ordered in pairs according to the of this is that the wave function in the node flux space behavior of quantum fluctuation in the Cooper pair con- is well localized and we can approximate the Josephson densate. Charge qubits with its single-charge tunneling is term to just second order, removing the anharmonicity. dual to flux qubits with single-flux tunneling, while phase qubits with phase oscillation are dual to the quasicharge The simplest realization of a phase qubit is a single qubits with quasicharge oscillations. These fundamentals Josephson junction qubit, which as the name suggests, qubit can be seen in Fig. 10. consists of a single Josephson junction with a current applied over it [73, 114]. The circuit can be seen in Fig. 11(a), where the parasitic capacitance, CJ , of the A. Phase qubits Josephson junction with energy EJ is drawn explicitly. A bias current is applied to the element, as indicated by I, The simplest superconducting qubits are called phase which fixes the anharmonicity. qubits, which is a current-biased Josephson junction, op- The source of the bias current can be thought of as a erated in the zero voltage state with a non-zero current very large inductor L, which is precharged with a very bias. They operate in the so-called phase regime, where large flux Φ such that I = Φ/L. Setting one of the plates EC EJ , i.e., the effective energy of the capacitor is of the capacitor to ground we are left with one node  31

CJ

EJ

Cg

Vg

Figure 12. Circuit diagram of the single Cooper pair box, con- Figure 11. (a) Circuit diagram of a single Josephson junction sisting of a Josephson junction, with energy EJ and parasitic qubit. On the top is the Josephson junction with energy EJ capacitance CJ , in series with a capacitor with capacitance Cg. and parasitic capacitance CJ . The element at the bottom The gate voltage is denoted Vg and the system is connected to denotes the source of the current I. The dot denotes the only the ground in the right corner. There is only one active node, node in the circuit, with flux φ through it. (b) Sketch of the denoted with a dot. washboard potential of a single Josephson junction qubit, for different values of the Josephson energy E and hence different values of the critical current Ic. The blue line has the largest 1. Single Cooper pair box anharmonicity as I is closest to Ic.

A significant improvement in coherence times in super- yielding the Lagrangian conducting qubits was found in 1997 with the discovery of the first charge qubit, known as the single Cooper pair 2 box [115]. The circuit consists of a Josephson junction CJ ˙2 (Φ φ) = φ + EJ cos φ − with energy E and a capacitor with capacitance C in L 2 − 2L J g series, with a superconducting island in between them. CJ ˙2 = φ + EJ cos φ Iφ, A parasitic capacitance CJ is included in the Josephson 2 − junction. This is a lumped circuit element representation where we have taken the limit L in the last line, of the natural capacitance that the junction will have by killing the quadratic term in φ and→ leaving ∞ Φ/L I (the way of construction. The circuit is biased with a gate constant irrelevant quadratic term in Φ is removed).→ The voltage Vg over the capacitor, which makes it possible Hamiltonian thus becomes to transfer electrons from the reservoir to the supercon- ducting island via the gate capacitance Cg. The circuit is 2 ˆ ˆ connected to the ground and thus there is only one active ˆ = 4EC qˆ EJ cos φ + Iφ. (194) H − node with flux φ through it. The corresponding circuit As the last two terms are the potential terms, we obtain diagram can be seen in Fig. 12. a washboard potential, as seen in Fig. 11(b). We follow the method presented in Section I B. In Due to this slope of the potential it is possible to drive order to write the Lagrangian, we must consider the fixed gate voltage. We model this as an external node with a the current I close to the critical current Ic of the Joseph- ˙ well-defined flux φV = Vgt, meaning φV = Vg. Setting son junction (see Eq. (13)), and thus introduce an an- T harmonicity. One adjusts the bias current I such that φ = (φ, φV ) we write the Lagrangian a suitable anharmonicity is introduced while minimizing 1 ˙ T ˙ the macroscopic tunneling through the right side of the = φ Cφ + EJ cos φ, (195) L 2 potential’s local minimum. The fact that we can tune the anharmonicity and spacing of the energy levels dynami- where the capacitance matrix is cally is a strength of this type of qubit. However, these   qubits have rather large decoherence noise compared to CJ + Cg Cg C = − . (196) the non-tunable charge qubits as introduced below. Cg Cg − ˙ Since we know that φV = Vg is a classical externally B. Charge qubits controlled variable, it should not be quantized. Therefore we only calculate the conjugate momentum ˙ Another kind of qubits is the so-called charge qubits. q = (Cg + Cj)φ CgVg. (197) These have their name from the fact that the basis states − of the qubit are charge states, meaning that they are Solving for φ˙ we do the Legendre transformation and find only dependent on the number of excess Cooper pairs the Hamiltonian in a disconnected superconducting island, and mostly 2 independent of the node fluxes. We will start from the 1 2 CgVg = (q + CgVg) EJ cos φ. (198) single Cooper pair box and move onto the transmon qubit. H 2(Cg + CJ ) − 2 − 32

CJ (a) EJ/EC = 0.0 (b) EJ/EC = 1.0 4 01 EJ

E/E 2

0 Cg (c) EJ/EC = 3.0 (d) EJ/EC = 10.0 CB 4 Vg 01

E/E 2 Figure 14. Circuit diagram of the transmon qubit, consisting of a Josephson junction, with energy EJ and parasitic capacitance 0 2 1 0 1 2 2 1 0 1 2 CJ , in series with a capacitor with capacitance Cg. The system − − ng − − ng is shunted by a large capacitance, CB . The gate voltage is denoted Vg and the system is connected to the ground in the Figure 13. The energies of the lowest lying states of the single right corner. There is only one active node. Cooper pair box/transmon qubit as a function of the bias charge ng. The difference between the two lowest bands are approximately equal to EJ at the avoided crossing. leads to the following term in the Hamiltonian

∞ EJ X ˆ = ( n n + 1 + n + 1 n ) . (202) We now change into conventional notation and define the J 2 H − n=−∞ | ih | | ih | effective capacitive energy

Solving the full system, ˆ = ˆC + ˆJ , (using either e2 H H H EC = , (199) Mathieu functions [116] or numerically) yields the avoided 2(Cg + CJ ) crossings seen in Fig. 13(b). The distance between these avoided crossings is approximately equal to the Josephson which means that we can write the Hamiltonian as junction energy EJ for the lowest states in the spectrum. In order to realize a qubit we set n equal to some half- 2 ˆ g ˆ = 4EC (ˆn ng) EJ cos φ, (200) H − − integer, which yields two states close to each other but with a large gap to higher states [112]. where we have quantized the dynamic variables and re- This qubits is, however, quite sensitive to small fluctua- moved constant terms. We have further defined the offset tion of the gate voltage Vg, since this changes ng, and the charge ng = CgVg/2e. energy dispersion is quite steep around the working point We can now discuss the operational regime of the ng = n + 1/2, as seen in Fig. 13(b) for EJ /EC = 1.0, Cooper pair box. When the Josephson junction is absent meaning that the qubit only works in this sweet spot, as it from the circuit (EJ /EC = 0), and thus absent from the is sensitive to charge noise. This reduces the decoherence Hamiltonian, the energy spectrum of the system is simply time of the system. The transmon qubit attempts to fix a set of parabolas when plotted against ng, one for each this problem. eigenvalue of nˆ. The parabolas cross at ng = n + 1/2, where n Z, see Fig. 13(a). If we consider the eigenstates of nˆ we find∈ that the states n and n + 1 are degenerate | i | i 2. Transmon qubit at ng = n+1/2. These states are essentially charge states of the capacitor. In this picture, the Hamiltonian of the capacitor becomes The transmission-line shunted plasma oscillation qubit (or just transmon qubit) was proposed in 2007 as an ∞ attempt to increase the coherence time in charge qubits X 2 ˆC = 4EC (n ng) n n , (201) [90, 117]. It exploits the fact that the charge dispersion H − | ih | n=−∞ reduces exponentially in EJ /EC , while the anharmonicity only decreases algebraically in EJ /EC , following a power which in matrix representation is just a diagonal matrix law. The setup resembles that of the single Cooper pair 2 with (n ng) on the diagonal. box, the difference being a large shunting capacitance, − Introducing the Josephson junction lifts the degeneracy, CB, between the two superconducting islands, followed thus making an avoided crossing at ng = n + 1/2. The by a similar increase in the gate capacitance Cg. The matrix representation now becomes a tridiagonal matrix circuit diagram is seen in Fig. 14. with EJ /2 on the diagonals below and above the main Capacitors in parallel simply add to one effective ca- diagonal, which is just the entries from the capacitor pacitor, hence the effective capacitance can be seen discussed above. The addition of the Josephson junction as the sum of the capacitance of the three capacitors 33

(a) (b) (c) CΣ = CJ + CB + Cg. With this elementary knowledge 1 1 the Hamiltonian of the transmon become identical to that EJ E of the single Copper pair box from Eq. (200), with the EJ J EJ ~Φ ~Φ EJ exception that EJ 2 ~Φ EJ EJ 2 E e 2 J EC = , (203) 3 2(CJ + CB + Cg) Figure 15. Circuit diagrams of different flux qubits. (a) The where we have simply changed the effective capacitance (symmetric) SQUID consists of two Josephson junctions in in Eq. (199). This gives much more freedom in choosing parallel (capacitances are not included as they are irrelevant the ratio EJ /EC , and we can thus solve the Hamiltonian here) (b) C-shunted flux qubit. Two Josephson junctions in for the energy dispersion for larger EJ /EC . The result is series are placed in parallel with a third Josephson junction. seen in Fig. 13(c) and (d). Both the parasitic and shunting capacitances have been in- From these results we observe that the energy dispersion cluded in the capacitance. (c) Fluxonium qubit. An array of becomes flatter for larger ratios of EJ and EC , which Josephson junctions have been placed in parallel with another means that the qubit becomes quite insensitive to charge Josephson junction, effectively creating an inductor in parallel noise, in fact a completely flat dispersion would lead to with the Josephson junction. no charge noise sensitivity at all. However, we also notice that the anharmonicity decreases for larger ratios. This is a result of the before mentioned fact that the charge As a response to external flux, currents occur in super- conducting materials in order to enhance or diminish the dispersion decreases exponentially in EJ /EC while the anharmonicity has a slower rate of change given by a total flux such that an integer number of flux quanta is power law. Therefore we cannot just increase the shunting achieved. capacitance until all charge noise disappear, as we still A superposition of clockwise and counterclockwise cur- need a working qubit. We are thus left with some effective rents is obtained by setting the external magnetic field values for the transmon which are usually somewhere in at half a magnetic flux quantum. Changing to node flux the range E /E [50, 100]. space this can be seen as a superposition of the ground J C states in a double well potential. In the double well po- Even though the∈ transmon has a ratio E /E close to J C tential a small tunneling occurs between the two sides of that of the phase regime (E E ), it is still classified C J the well, which couples the two wave functions, making as a charge qubit due to the structural similarity to the an avoided crossing, and thus a two level system close to single Cooper pair box qubit. Due to that and the fact each other, but with a very large gap to the remaining that capacitors in parallel add, we will often just put a states. Josephson junction and a parasitic capacitance in place of the transmon in larger circuits for simplicity. We further notice that if the ratio is large enough, the bias voltage 1. DC-SQUID becomes irrelevant and can be omitted as well. When implementing the transmon qubit on an actual chip, an Xmon qubit is often used [44]. The Xmon qubit A superconducting quantum interference device, or just is essentially the same as that of a transmon, in fact its SQUID, consists of two Josephson junctions in a ring, ˜ electrical circuit model is equivalent to that of a transmon, with an external flux, Φ, going though the ring. A circuit however, when implemented in an actual chip the Xmon diagram of a SQUID can be seen in Fig. 15(a). If we provides more possibilities for couplings, due to its design add an external bias (DC) current over the SQUID, from of having cross-shaped arms (hence the name). node 1 to node 2, we obtain a DC-SQUID [119]. We Recently a dual to the transmon qubit, called a qua- consider symmetrical junctions in the SQUID, but it is a sicharge qubit, or blochnium, have been proposed where neat exercise to extend it to asymmetrical junctions. the shunting capacitance is replaced by a large shunt- Choosing the spanning tree over the capacitors [not in- ing inductance. This large inductance makes the qubit cluded in Fig. 15(a)] the potential part of the Hamiltonian very robust against flux noise, which could open up for becomes exploration in high-impedance circuits [118]. ! ! Φ˜ Φ˜ U = EJ cos φ1 φ2 + EJ cos φ2 φ1 + , − − 2 − − 2 C. Flux qubits (204) where we have divided the flux equally between the two arms of the SQUID. Rewriting this using the trigonometric Flux qubits are implemented in a looped superconduct- identity 2 cos α cos β = cos(α β) + cos(α + β) we find ing circuit interrupted by one or more Josephson junctions. − A current is driven through these circuits using the fact ˜ ! that the fluxoid quantization means that only an integer Φ U = 2EJ cos cos φ, (205) magnetic flux quanta is allowed to penetrate the loop. − 2 34 where we have defined φ = φ1 φ2. Note that the fluxoid quantization condition gives us− a constraint on the fluxes, namely

φ1 φ2 + Φ˜ = 2πk, (206) − where k is an integer, meaning that we can remove one degree of freedom. In other words we have obtained an 0 ˜ ˜ effective Josephson energy of EJ (Φ) = 2EJ cos(Φ/2) , which means that it is possible to dynamically| tune the| Josephson energy. This idea is often implemented in superconducting circuits instead of a single Josephson junction so that the spacing of the energy levels can be tuned dynamically by changing EJ via Φ˜. However, we will usually just place a single Josephson junction in a Energy circuit diagram.

2. C-shunted flux qubit

The idea behind the C-shunted flux qubit (CSFQ) is the same as for the transmon, however here the capac- itive shunting is over a flux qubit (Sometimes called a persistent-current qubit (PCFQ)) [120, 121]. As with the transmon qubit the C-shunting improves the coherence Figure 16. Spectrum of the fluxonium qubit at the two different of the qubit [43, 122]. We therefore consider the flux flux biasing points. For this plot the parameters have been set qubit without going in to details of the shunting, see to EC /h = 1 GHz, EJ /h = 3.43 GHz and EL/h = 0.58 GHz. Section III B 2. The flux qubit consists of two Josephson junctions This potential no longer has the usual sinusoidal form, in series, with energy γE , which are then placed in J and its final form depends on the external flux Φ˜ and parallel with a third Josephson junction, with Josephson the junction ratio γ. The most common configuration is energy E , where γ is ratio of the geometrical size of the J for an external flux Φ˜ = (1 + 2l)π, where l Z. These Josephson junctions. The qubit can be seen in Fig. 15(b), points are often called the flux degeneracy∈ points and where all parasitic and shunting capacitances are collected corresponds to one half superconducting flux quantum in a single capacitor. Naively it looks like there are three threading the qubit loop (remember that there is a factor degrees of freedom in this circuit (or at least two if the of Φ /2π on the external flux). In this configuration the ground node is removed), one for each node. However, to 0 qubit frequency is most robust against flux noise, leaving a good approximation it turns out that there is only one the qubit with optimal coherence times. true degree of freedom when we assume γ > 1. This is due to the fact that the node in between the two Josephson As mentioned above we have assumed γ > 1, which junctions is a passive node. Using the same trigonometric means that we could eliminate a degree of freedom. This tricks as for the SQUID, we can write the potential energy can be understood as approximating the particle as mov- of the three Josephson junctions as ing in one dimension in a two-dimensional potential, and is sometimes called the quasi-1D approximation, this ap-     proximation fails if γ < 1. If 1 < γ < 2, the potential ψ+ ψ− ˜ U = EJ 2γ cos ψ2 cos + cos(ψ− + Φ) , takes the form of a double well, which has been inves- − 2 − 2 (207) tigated as the PCFQ [120, 121]. If, on the other hand, γ > 2, the potential takes the form of a single well much where we have introduced the change of coordinates ψ± = similar to the transmon qubit, which is why the CSFQ has φ1 φ3 and ψ2 = φ2 which is the middle coordinate in between± the two Josephson junctions. This coordinate been investigates in this regime [43, 122]. In both cases, transformation turns out to diagonalize the capacitance if the anharmonicity is sufficiently large, the quantized potential can be truncated to the lower levels. matrix as well leaving only ψ− a non-zero eigenvalue, thus the two remaining node fluxes must be superfluous, and from the constraints obtained from the Euler-Lagrange 3. Fluxonium equations we find that ψ2 = ψ+/2, which leaves the potential The fluxonium qubit is the natural extension of the   ψ− flux qubit. Instead of two Josephson junctions in parallel U = EJ 2γ cos + cos(ψ− + Φ)˜ . (208) − 2 with another Josephson junction the fluxonium features 35

an array of up to N = 100 Josephson junctions [123–125], 1 sometimes referred to as a kinetic or super inductance. The circuit diagram can be seen in Fig. 15(c). Using L EJ,CJ the same quasi-1D approximation as in Section III C 2 C repeatedly, we arrive at a potential C 4 2 ~  ψ  Φ U = EJ Nγ cos + cos(ψ + Φ)˜ , (209) EJ,CJ L − N where ψ is the sum of all node fluxes in between the array 3 of Josephson junctions on the left side of Fig. 15(c). When the number of Josephson junctions N becomes large the Figure 17. Circuit diagram of the 0-π qubit. Four nodes are argument in the first cosine, ψ/N, becomes small such connected each other by two large superinductors (drawn here as regular inductors) L, two Josephson junctions EJ with that the cosine can be approximated by a second order parasitic capacitance CJ , and two shunting capacitors C. approximation which leaves

1 2 ˜ U = ELψ EJ cos(ψ + Φ), (210) have a capacitance of CJ , and the superinductors as L. 2 − Superinductors are usually made as an array of Josephson where EL = EJ γ/N. This has the same effective form junctions (see Section III C 3), however, here we draw as an rf-SQUID [126], however the kinetic inductance of them as regular inductors as this is their effective form. the fluxonium qubit is much larger than the geometric An external flux, Φ˜, goes through the qubit. It is advan- inductance of the rf-SQUID. When the external flux bias tageous to choose the spanning tree such that only the is Φ˜ = 0 the potential has minimum at ψ = 0. For small Josephson junctions lie in the set of closure branches. fluctuations of ψ the potential is approximately harmonic The node fluxes of the circuit are denoted and the lowest lying states are close to simple harmonic (φ1, φ2, φ3, φ4), and the normal modes of the circuit can oscillator states. At higher energies the anharmonic cosine be written using the transformation term of the potential comes into play as seen in Fig. 16. ϕ  1 1 1 1  φ1 This ensures the anharmonicity necessary for using the − − θ 1 1 1 1 1 φ2 two lowest lying states as the qubit subspace. However,   = − −    . (211) ˜ ζ  2  1 1 1 1 φ3 the fluxonium qubit is most often operated at Φ = π, − − similarly to the flux qubit. In this regime the potential Σ 1 1 1 1 φ4 exhibits a double well structure, and it is possible to Here Σ is the CM coordinate, which has no influence on achieve a much larger anharmonicity than in the Φ˜ = 0 the dynamics of the system and can be discarded. This case. basis transformation diagonalizes the capacitance matrix In recent experiments fluxonium qubits have reached C = 2 diag(C ,C + C,C). The Hamiltonian then takes impressive lifetimes of 100 400 s [125]. This is done J J µ the form while maintaining a large anharmonicity− suitable for fast gate operation. This puts fluxonium among the top qubit 2 2 2 EL 2 2 =4ECϕn + 4ECθn + 4ECζ n + (ϕ + ζ ) candidates for near future applica- H ϕ θ ζ 2 tions. In addition, the success of the fluxonium qubit h i EJ cos(θ + ϕ) + cos(θ ϕ Φ)˜ , proves that long coherence times can be achieved even − − − in more complicated system with a large number of spu- (212) rious modes. This should encourage quantum engineers −1 to further explore circuit design utilizing large kinetic where nϕ, nζ , nθ are the canonical momenta, ECϕ = 16CJ , inductance. −1 −1 ECθ = 16(C + CJ ) and ECζ = 16C are the charging ener- 2 gies of each mode, while EL = L is the effective inductive energy. Note that the ζ-mode completely decouples from 4. 0-π qubit the rest of the system and can thus be ignored. By trans- Φ˜ forming the θ variable θ θ + 2 we can rewrite the A new type of qubit is the 0-π qubit. It has been Hamiltonian into the simpler→ form proposed more recently than the above qubits, but it shows promising tendencies in topological protection from 2 2 =4ECϕnϕ + 4ECθnθ noise [127–133]. H ˜ ! (213) The 0-π qubit consists of four nodes which are all con- Φ EL 2 2EJ cos θ cos ϕ + ϕ . nected to each other by two large superinductors, two − − 2 2 Josephson junctions, and two large shunting capacitors, as shown in Fig. 17. We denote the shunting capacitors The circuit is engineered such that C CJ , and we can  as C, the Josephson junctions as EJ and assume they thus think of the system as a heavy particle moving along 36

couplings where couplings tuned using external fluxes are introduced [136–138]. Coupling of qubits naturally increases the complexity of the system, which is why the following examples are considered more difficult than the above.

1. Gmon coupler

The gmon coupler introduces tunable swapping cou- plings between two transmon-like qubits by exploiting mutual inductance [135]. Two transmon qubits are cou- pled via a Josephson junction of energy Ec = 1/Lc, where Lc is the Josephson inductance as in Eq. (14), and in parallel with each qubit is a linear inductor of inductance Lg. The circuit diagram can be seen in Fig. 19(a). The crucial point of the gmon coupler is that the two linear inductors have a mutual inductance between them. Figure 18. Wave functions of (a) the ground state and (b) first excited state for the 0-π-qubit. The contour lines indicate An excitation current Iq over, say, the right qubit (both the qubit potential. For this plot the parameters were set to the Josephson junction and the capacitor) flows through EC,θ = 0.1, EC,ϕ = 10, EJ = 10 and EL = 1. both Lg and Lc, and Kirchhoff’s current law gives us Iq = IL1 + IJc. Using Kirchhoff’s voltage law going around the inner loop (in the direction indicated by the ˙ the θ-axis and a lighter particle moving along the ϕ-axis. arrow) of the circuit over the inductors we find LcIJc + In the basis of the computational states 0 and 1 (which LgI˙L2 = LgI˙L1. Note that this assumes no direct mutual are chosen as the ground and first excited| i state| i respec- inductance between the inductors. However, assuming tively) the θ variable is well localized around either 0 or π, that the current over the second qubit is negligible, we as shown in Fig. 18, which is also the reason for the qubit’s can integrate this from minus infinity to t and insert our name. Setting θ = 0 or π in Eq. (213) we see that the result from Kirchhoff’s current law to find potential along the ϕ-axis is similar to that of fluxonium biased by a flux of either 0 or π. As a result the two states Lg n n IJc = Iq , (214) have vanishing matrix elements 0 θ 1 , 0 ϕ 1 0. Lc + 2Lg This makes the qubit highly resistanth | | toi noise-inducedh | | i ' relaxation as described in Section II C. i.e., we have the fraction of current going through the In recent experiments [132] with the 0-π qubit, relax- coupler. This current generates a flux in the second ation times above 1 ms have been achieved, making it an qubit Φ2 = LgIJc = MIq, where we have expressed the exciting candidate for future research. As with fluxonium, flux using an effective mutual inductance M. Solving the 0-π qubit proves that it is not only the most simple for M and inserting Eq. (214) we find that this mutual qubits, such as the charge and flux qubit families, that can inductance is achieve long coherence times. Researchers should make note of this when developing new circuit designs in order L2 M = g . (215) to tap into the potential that more complicated compo- 2L + L nents, such as the kinetic inductances used in fluxonium g c and the 0-π qubit, bring to the table. Now the inductance of the coupling Josephson junction is actually dependent on the Josephson phase, Φc, of the coupler, which is why we must exchange Lc Lc/ cos Φc, D. Coupled qubits meaning that we can tune the mutual inductance→ by changing the Josephson phase. In the above examples we presented a few different qubit The next step is to find the Hamiltonian. As seen above, designs, and the next step is naturally to try to couple the effect of the Lc junction can be modelled as a mutual these. The simplest form of coupling is to couple two inductance. We will therefore use the method presented qubits using a capacitor or inductor like in the examples in Section II B. To begin with we ignore dissipation. We in Figs. 1 and 8. This yields a simple linear coupling therefore label all seven branches in Fig. 19 corresponding like in Eq. (75) [134]. We therefore focus on couplings to the circuit graph. We choose our spanning tree as the beyond static linear couplings. Below we present two capacitive subgraph of the circuit (see Section II A). This tunable couplers [107, 135], but there are other types of means that all inductive elements lie in the set of links. 37

Lc Using Eq. (141) we finally find the Hamiltonian of the circuit to be I Jc 1 1 = QT C−1Q L [cos Φ + cos Φ ] + ΦT L0−1Φ, 2 J 1 2 2 C IL2 I C H − L1 Iq (222) LJ LJ L L where we have removed the φc term as it is effectively g g included in the mutual inductance, such that the branch T T fluxes and currents are given as Φ = (Φ1, Φ2) and Q = (Q1,Q2) respectively. If we only consider the interacting part of the Hamiltonian, which is only in the inductive Figure 19. Circuit diagram of the gmon coupler. Two term, we find transmon-like qubits (C,EJ ) are coupled via a Josephson junction of inductance LJ . A linear inductor of inductance M Lg is in parallel with the qubits leading to mutual inductance int = Φ1Φ2. (223) H −L2 M 2 between the inductors, which allows for the tuning of the g − coupling. Quantizing the Hamiltonian and changing into step oper- ators this term becomes In terms of the current vectors, we have [see Eq. (116)] ˆ 1 ζM ˆ†ˆ ˆ ˆ† int = 2 2 (b1b2 + b1b2), (224) IL1 H −2 L M g −   IL2 IC1   It = , Il = IJ1 , (216) where we have changed into step operators [Eq. (52)] and IC2   IJ2 employed the rotating wave approximation in the second IJc equality, and the impedance, ζ, is given by Eq. (50). + − Changing to a qubit model we can write int = g(σ1 σ2 + where we note that in this notation we have Iq = IJ1 +IC1. − + H σ1 σ2 ) (see Section I F 1), with the coupling constant Using the definitions in Section II A 1 we find the F-matrix being as  1 0 1 0 1 ζ 1 F = . (217) g = , (225) −0 1 0 1− 1 −2 2Lg + Lc/ cos Φc − As a sanity check we use Eq. (118) combined with where we have assumed Lg M.  Eq. (121) to find the same equations as Kirchhoff’s current We now want to include dissipation as in Section II C. law in Section II B 1 would yield. We do this by adding a single impedance, which we place The reduced linear inductance matrix with all non- over the coupling Josephson junction, such that it affects inductive entries removed, as used in Eq. (128), is both degrees of freedom. This means that we must add the following matrix to Eq. (217)   0 Lg M L = , (218)   MLg 1 m¯ = , (226) −1 which we can expand to the full linear inductive matrix by adding a row and a column of zeros such that which is exactly the vector we need in order to model dissi- L0 0 pation via the Caldeira-Leggett model Section II C 4. We L = . (219) 0 0 must now determine the dissipation matrix in Eq. (153). We find that µ = 2, and K(ω) = iω/Z(ω), where the −1 impedance Z(ω) should be determined experimentally The Josephson matrix is LJ = diag(EJ ,EJ ,Ec). Com- bining all this we find that the equations of motion from and then modelled using the spectral function in Eq. (144). Eq. (133) In order to progress any further we need actual experi- mental values of the components. We therefore leave the CΦ¨ = L−1sinΦ L+Φ, (220) example here. − J − T where C = diag(C,C, 0), the flux Φ = (Φ1, Φ2, Φc), and   2. Delft coupler Lg M 0 + 1 − L =  MLg 0 , (221) L2 M 2 − The Delft coupler [107] introduces tunable non-linear g − 0 0 0 couplings between two qubits in a center-of-mass-basis. where the non-zero part of the matrix is the inverse to The following example is a simplification of the supple- Eq. (218). mentary material to Ref. [107]. 38

(1) c (2) EJ EJ EJ and quantizing we find the Hamiltonian

ˆ 1 T −1 (1) ˆ (2) ˆ = p K p EJ cos(√2ψ1) EJ cos(√2ψ2) 1 2 3 4 H 2 − − ˆ ˆ ! c ψ1 ψ2 ˆ E cos − ψS , − J √2 − Cg C Cc C Cg (231) where p is the vector of conjugate momentum of the ψ C C g g vector. Expanding the cosines we arrive at the following Hamiltonian and changing into step operators [Eq. (52)] Figure 20. Circuit diagram of the Delft coupler. Two the non-interacting Hamiltonian takes the form transmon-like qubits (1 and 2) are coupled via another X h ˆ†ˆ αi ˆ†ˆ†ˆ ˆ i transmon-like coupler (c). ˆ0 = ωib bi + b b bibi , (232) H i 2 i i i={S,1,2}

where we have defined ωi = 4√EC EJ + αi and αi = Consider the circuit diagram in Fig. 20. Following the 2 (1) approach in Section I B we find the following capacitance ζ (EJ + 3EJ )/8, with the effective capacitive energies − (i) −1 matrix being the usual EC = (K )(i,i)/8, which turns out to be the same for the 1 and 2 mode, thus we call it simply (1) (2) C + Cg C 0 0  E = E = E . We have also defined the effective − C C C CC + Cg + Cc Cc 0 (1) c C =  − −  , Josephson energy EJ = EJ + EJ /4 and assumed that  0 Cc C + Cg + Cc C  − − the 1 and 2 modes are resonant, i.e., E(1) = E(2). Lastly 0 0 CC + Cg J J − we have also defined the impedance, as in Eq. (50), as (227) p where we have defined the flux vector as φ = ζ = 4EC /EJ . We do not include the center-of-mass T coordinate as it does not influence the dynamics of the (φ1, φ2, φ3, φ4) . This yields the following circuit La- grangian system. Note how the two modes are affected by both their ’own’ Josephson junction and the coupling Josephson junction. 1 ˙ T ˙ (1) = φ Cφ + E cos(φ1 φ2) Assuming that the sloshing mode, ψ , is detuned from L 2 J − (228) S c (2) the remaining two modes we can remove couplings to + E cos(φ2 φ3) + E cos(φ3 φ4). J − J − the sloshing mode using the rotating wave approximation from Section I E. After this approximation the interaction We now change into a CM-basis (see Section I B 3) of the part of the Hamiltonian takes the form capacitive sub-system using the following transformations ˆ ˆ ˆ† V ˆ†ˆ ˆ†ˆ V ˆ†ˆ†ˆ ˆ I =Jb1b2 + b1b1b2b2 + b1b1b2b2 H 2 4 (233) 1 2V ˆ ˆ† ˆ ˆ† ψ = (φ + φ + φ + φ ), (229a) + (b1nˆ1b2 + b2nˆ2b1) + H.c., CM 2 1 2 3 4 3 1 where we have used the assumption that the modes are ψ1 = (φ1 φ2), (229b) √2 − resonant. The swapping coupling strength is given by 1 1 ζEc 2V ψ2 = (φ4 φ3), (229c) J = (K−1) J (234) √2 − 2ζ (1,2) − 4 − 3 1 ψS = (φ1 + φ2 φ3 φ4). (229d) The non-linear coupling factor is given as 2 − − Ec ζ2 V = J . (235) This decouples the center-of-mass coordinate, ψCM , from − 16 the remaining coordinates (note that this is due to the The first non-linear term in Eq. (233) is sometimes called identical grounding capacitances Cg) as the transformed the cross-Kerr coupling term with coupling strength V , capacitance matrix takes the form while the second non-linear term tunnels a pair of excita- tions from one mode to the other with coupling strength   2Cg 0 0 0 V/4. Therefore this term does not contribute if the Hamil- 1  0 4C + 2Cg + Cc Cc √2Cc  tonian is truncated to a two-level model, but it may result K =  − −  , 2  0 Cc 4C + 2Cg + Cc √2Cc  in correction to the model (using for instance the method − 0 √2Cc √2Cc 2Cg + 2Cc described in Section I F 3). Thus truncating to a two-level − (230) model the Hamiltonian becomes where we have chosen the basis such that ψ = + − − + V z z T ˆ = ˆ0 + J(σ σ + σ σ ) + σ σ . (236) (ψCM , ψ1, ψ2, ψS) . Doing a Legendre transformation H H 1 2 1 2 4 1 2 39

CJ where the capacitance matrix is   C + CJ CJ C a b − − C =  CJ C + CJ C  , (238) − − C C 2C + CJ ~ ~ − − Φ Φ 1 Eq 2 T C C and we have defined q = (qa, qb, qc). We now follow E L L' E Section I B 3 and transform into a CM system of the J J capacitive sub-system using the following transformation c 1 ψ1 = (φa φb), √2 −

CJ Eq 1 ψ2 = (φa + φb 2φc), (239) √6 − 1 ψCM = (φa + φb + φc). √3 Figure 21. Circuit diagram of two coupled qutrits. The three From this the transformation matrix V can be constructed circuit nodes, φa, φb, φc, are indicated by dots. such that Eq. (34) is satisfied. With this transformation the capacitance matrix takes the form E. Coupled qutrits 2C + C 0 0  J √ K = 0 2 C + 3C 2 C , (240) In this example, we present a system with two qutrits  3 J√ 3 J  2 −1 coupled via an effective Heisenberg XXZ-coupling. The 0 CJ CJ − 3 3 proposed implementation is shown in the effective lumped where is the transformation defined by Eq. (239) when circuit element model in Fig. 21. The idea is to mix V T the basis is chosen such that p = (p1, p2, pCM ) as in the nodes φa, φb and φc such that we obtain two low- dimensional degrees of freedom (after truncation) with Eq. (37). Assuming that K is invertible its inverse be- the desired coupling and a decoupled third degree of comes freedom which can be seen as a center of mass coordinate  1 0 0  and can be ignored. Due to the left-right and top-down 2CJ +C √ K−1 =  0 1 2  , (241) symmetry there is no resulting exchange interaction from  √3C 3C  0 2 3 + 2 the capacitances and the Josephson junctions. By picking 3C CJ 3C an asymmetry in the inductances, we can engineer the We notice that the diagonal terms for ψ and ψ are un- system to obtain the desired exchange interaction, and 1 2 equal, which becomes important when we later introduce it can be completely eliminated by making L = L0. On bosonic step operators related to the harmonic part of the other hand, the longitudinal ZZ-coupling does not the full Hamiltonian. cancel in the qutrits, giving an energy shift depending on Returning to the potential part of the Hamiltonian in the other qutrit state, thus removing all degeneracies in Eq. (237) we rewrite it in the CM-basis in Eq. (239) the two-qutrit system. This interaction can furthermore and apply the standard procedure of rewriting using be tuned by adding external fluxes. Also, we include trigonometric identities as in Section III C 1, and requir- driving lines to each of the three nodes, which enables ing Φ˜ = Φ˜ = Φ˜ . Finally we expand the cosine terms us to control the qutrit energies dynamically by the AC 1 2 to fourth order,− assuming that we are in the transmon Stark shift arising from detuned driving as explained in regime. From here on, we discard fast rotating terms, Section I G. We note that the inductances in Fig. 21 may as they will vanish when employing a rotating wave ap- be physically arranged in a manner that may allow for a proximation later anyway, see Section I E. After the dust mutual inductance as an additional manner of coupling. settles, we arrive at the potential part of the Lagrangian This could be analyzed as above int he Gmon case, and in the transformed variables we neglect this here for simplicity.   We start from the circuit in Fig. 21, which yields the 2 1 EJ 4 U(ψ) E1ψ 4Eq + cos Φ˜ ψ following Hamiltonian using the method of nodes as pre- ' 1 − 24 2 1 sented in Section I B   2 1 4 9 4 + E2ψ Eq + EJ cos Φ˜ ψ (242) 1 T 1 2 1 2 2 2 = q Cq + (φa φc) + (φb φc) − 24 9 2 H 2 2L − 2L0 − 3EJ ˜ 2 2 E (cos(φ φ ) + cos φ ) gψ1ψ2 cos Φψ ψ , q a b c − − 8 1 2 − h − i EJ cos(φa φc + Φ˜ 1) + cos(φb φc + Φ˜ 2) , − − − we have neglected all energy and coupling terms involving (237) ψCM as the ψCM -degree of freedom will typically have an 40

2 2 energy spectrum far from the rest. We have also removed 1 2 all non-energy-conserving terms. We have defined the ω2,2 effective energies and coupling ω1,2

EJ 2 E =E + E 0 + cos Φ˜ + E , (243a) 1 L L 2 q   1 EJ 1 ω2,1 E =3 E + E 0 + cos Φ˜ + E , (243b) ω1,1 2 L L 2 9 q

g =2√3(EL EL0 ), (243c) − 1 2 0 where EL = 1/4L and EL0 = 1/4L are the effective inductive energies. Note the asymmetry between the Figure 22. Energy diagram of the system of two qutrits 1 and 2 mode. Ignoring the CM-mode we quantize the described by the Hamiltonian in Eq. (246) resulting from the Hamiltonian and change into step operators using Eq. (52), circuit in Fig. 21. The qutrits are connected with a Heisenberg p with the impedances ζi = 4EC,i/Ei, where EC,i is the XXZ-coupling seen in Eqs. (248) and (250). usual effective capacitive energy. The Hamiltonian takes the form Note that there is also a 11 20 term, however, this is 2 suppressed due to the anharmonicity| ih | and thus we can X h p ˆ†ˆ αi ˆ† ˆ 4i ˆ = 4 EC,iEib bi + (b + bi) H i 12 i removed it using the RWA from Section I E. Finally the i=1 longitudinal interaction part of the Hamiltonian is 1 p ˆ† ˆ ˆ† ˆ (244) + g ζ1ζ2(b + b1)(b + b2) z z 1 2 ˆZ = JZ S S , (250) 2 H 1 2 3EJ ˆ† ˆ 2 ˆ† ˆ 2 z cos Φ˜ζ1ζ2(b + b1) (b + b2) , where S = 0 0 + 3 1 1 + 5 2 2 , which is a general- − 32 1 2 | ih | z| ihz | | ih | ization of the qubit σ1 σ2 longitudinal coupling. where we have defined the anharmonicities 2   ζ1 EJ 1. External driving of the modes α1 = 4Eq + cos Φ˜ , (245a) − 8 2 2   ζ2 4 9 We wish to control the two modes using external mi- α2 = Eq + EJ cos Φ˜ . (245b) − 8 9 2 crowave driving following the procedure presented in Sec- tion I G 1. We drive the three original modes, φa, φb, and We now wish to truncate to a three level system, also φc, of the system. This gives the three additional terms known as a qutrit, following the procedure presented in for the Lagrangian Section I F 2. We choose the zero point energy to be 2 2 the 0 0 states. This is contrary to the qubit where we Ca  ˙  Ca  ˙  ext = φa Va(t) + φb Vb(t) usually| ih chose| the zero point energy to lie in between the L 2 − 2 − 2 (251) two states. The diagonal part of the Hamiltonian becomes Cc  ˙  + φc Vc(t) , 2 2 − X ˆ0 = (ωi,1 1 1 + (ωi,1 + ωi,2) 2 2 ) , (246) where Vi(t) is the external microwave driving. H | ih |i | ih |i i=1 We want to effectively couple the ψ1 and ψ2 modes to (independent) external fields. Because these are specific where the energies are given as linear combinations of of φa, φb and φc, the couplings p ωi,1 =4 EC,iEi + αi, (247a) should be obtainable by coupling to these fields in the cor- rect way and rewriting the system a little. After rewriting ω =ω + α , (247b) i,2 i,1 i the terms in Eq. (251) a bit using the transformation in from which we clearly see the effect of the anharmonicity. Eq. (239), we can write this as the following requirements The transversal interaction part, which swaps excitation between the two qutrits, is ext Va(t) + Vb(t) = 2A1 cos(ω t), (252a) − − 1 X =JX ( 01 10 + 2 12 21 ) r H | ih | | ih | (248) ˙ ˙ 2Cc ˙ ˙ φa + φb φc = ψ2, (252b) + 2JZ 02 20 + H.c., | ih | − Ca r where the coupling constants are given as 2Cc V (t) + V (t) + V (t) = 0, (252c) a b C c 1 p a JX = g ζ1ζ2, (249a) r 2 2Cc ext Va(t) Vb(t) + Vc(t) = 4A2 cos(ω2 t), 3EJ − − Ca − JZ = cos Φ˜ζ1ζ2. (249b) − 32 (252d) 41

ext where ωi is the external driving of the ith qubit. Note that we do not include a phase in the driving for simplic- ity. From Eq. (252b) we quickly realize that Cc = 2Ca, which leaves three equations with three unknowns. These equations can be solved EJ

ext ext Va(t) = A1 cos(ω1 t) + A2 cos(ω2 t), (253a) E E ext ext J J Vb(t) = A1 cos(ω t) + A2 cos(ω t), (253b) − 1 2 ext Vc(t) = A2 cos(ω t), (253c) − 2 EJ and inserted back into ext. Expanding and collecting the terms, and ignoring irrelevantL offset terms independent of the phases, this leads to a total kinetic Lagrangian

2 1 ˙ T ˙ Ca X ext ˙ kin = ψ K˜ ψ Ai cos(ω t)ψi, (254) Figure 23. An example of a simple circuit with four nodes. L 2 − 2 i i=1 The nodes are connected in a square with Josephson junctions and a capacitor, and two of the nodes are connected to the opposite corner. where K˜ = K + Kext is the adjusted capacitance matrix T in the CM frame, where Kext = V CextV is the con- tribution from the coupling to the external nodes with yields a capacitance matrix Cext = diag(Ca,Ca, 2Ca). Performing a Legendre trans- formation and changing into step operators, the driving   term takes the form 2C + Cd C Cd C C −2C − C −0 C =  − −  . (257) 2 3  Cd C 2C + Cd C X Ca ext X −1 ˆ† ˆ − − − ˆext = i Ai cos(ω t) (K˜ ) (b bj). C 0 C 2C H 2√2ζ i (j,i) j − − − i=1 i j=1 (255) To begin with we set the diagonal capacitance Cd = C, We now define the Rabi frequency Ωi = which yields the modes P3 ˜ −1 Ca j=1(K )(j,i)/2√2ζi. Truncating to the low- est three states the driving term takes the form 1  1  1 1 1 0 vCM =   , v1 =   , 2   2 1 √2  1 ˆ X ext √ − ext = i Ωi cos(ωi t) 0 1 i + 2 1 2 i + H.c. 1 0 H | ih | | ih | (258) i=1     (256) 0 1 1 1 1 1 Depending on which transition we want to drive we must v2 =   , v3 = −  , match the driving frequencies with the transition energy, √2  0  2  1  ext 1 1 e.g., ω1 = ω1,2 if we want to drive the 1 2 transi- − − tion of the first qutrit, see Fig. 22. | i ↔ | i Such a system can, besides arbitrary one-qutrit gates with eigenvalues λCM = 0 (as we have not grounded and generalized controlled-NOT gates, implement both a any node), λ1 = λ3 = 4C, and λ2 = 2C. Note how the single-qubit and two-qubit non-adiabatic holonomic gates choice of identical capacitances ensures that the vi are [139, 140]. independent of the capacitances. These modes correspond to charge oscillating across the diagonal between nodes 1 and 3 (v1), oscillation across the sides of the square between nodes 2 and 4 (v2), and F. Multi-body interactions finally an oscillation involving the entire circuit between the nodes 1 and 3, and the nodes 2 and 4 (v3). We can The smallest example of multi-body interaction must see that changing the capacitance of the diagonal branch consist of four nodes, as we can always decouple the CM does not disturb the eigenmodes. The fact that this is mode leaving three true degrees of freedom. Consider the only branch where the capacitance can be changed is the circuit in Fig. 23. If we approximate the Josephson an intuitive result when we consider the modes in terms junctions as linear inductors, we quickly realize that the of oscillating charge. We can think of the mode v1 as the capacitive and inductive matrix can be diagonalized si- only one involving the diagonal branch. multaneously. We therefore first consider the capacitive Changing the diagonal capacitance to Cd = C, we only subnetwork, which, following the method in Section I B change the first mode, and the diagonalized6 capacitance 42 matrix takes the form (see Section I G), we must employ several control lines in order to couple to the eigenmodes. For concreteness   0 0 0 0 we consider the above circuit in Fig. 23 transformed to 0 2(C + C ) 0 0 K =  d  . (259) its eigenmodes. We now want to capacitively couple the 0 0 2C 0  ψ1 degree of freedom to an external control line, without 0 0 0 4C coupling to the two remaining degrees of freedom. We therefore couple each node in the (non-transformed) From this point on we remove the center of mass coor- circuit via identical capacitors of capacitance C to dinate. Consider now the inductive subnetwork of the ext external micro wave driving V (t). This results in the circuit in Fig. 23, it yields the potential energy i following additional terms in the Lagrangian " ψ ψ  1 2 4 2 U(ψ) = EJ cos − + ψ3 Cext X  ˙ ˙  − √2 ext = φi Vi . (262) L 2 −   i=1 ψ1 + ψ2 + cos ψ3 √2 − Writing this in terms of the electrical modes, expanding   the parenthesis and throwing away constant terms, we ψ1 ψ2 (260) + cos − ψ3 have √2 − # ψ + ψ  Cext h ˙ 2 ˙ 2 ˙ 2 ˙ 2 1 2 ext = ψ + ψ + ψ + ψ + cos + ψ3 2 CM 1 2 3 √2 L ˙   ψCM V˙1 + V˙2 + V˙3 + V˙4 Ed cos(√2ψ1), − − ˙   + √2ψ1 V˙1 + V˙3 (263) where we have changed to the diagonal basis. With some − algebra this can be reduced to ˙   + √2ψ2 V˙2 + V˙4 − U(ψ) = E cos(√2ψ ) ˙   d 1 + ψ3 V˙1 + V˙2 V˙3 + V˙4 . −     − − ψ1 ψ2 (261) 4EJ cos cos cos(ψ3). − √2 √2 We now want to only couple to ψ1, so we choose V˙1 = V˙3 − and V˙2 = V˙4 = 0. This yields From this we see that the diagonal Josephson junction, with E , becomes non-interacting because there is a mode d Cext h ˙ 2 ˙ 2 ˙ 2 ˙ 2 ˙ i ext = ψ + ψ + ψ + ψ 2√2ψ1V˙1 (264) corresponding exactly to this branch. We also see that L 2 CM 1 2 3 − the remaining four Josephson junctions with E create a three-body interaction. When expanding the cosines in The four first terms simply contribute to the diagonal of the interaction we get both corrections to the frequencies K, and can be viewed as corrections to the energy of the and two- and three-body couplings, which are the only modes, and the last term is exactly an interaction term interactions in the system. Note that the diagonal branch between ψ1 and the external V1(t). Note that we could can be removed from the system without changing the have simply coupled to the 1 and 3 node in order to get dynamics of the system. the same result, which we could have guessed from v1 in By introducing external fluxes, we can tune the triple- Eq. (258). Also note that the center of mass mode now cosine term to involve odd terms as well. The cosine terms have a non-zero eigenvalue, which is due to the fact that themselves only result in products of even powers of the all the nodes are now coupled to the ground. ψi’s, but with flux threading the circuit a cosine could even be tuned into a sine, making the contributions from the corresponding mode completely odd. This opens up IV. SUMMARY AND OUTLOOK the possibility for further multi-body couplings achieved through the normal modes. In this tutorial we have presented various methods used when analyzing superconducting electrical circuits. We have summarized the methods in Fig. 24, where we have 1. External coupling to eigenmodes included both the basic method and more advanced al- ternative methods. In general it is possible to apply the If we wish to couple an eigenmode-circuit into a larger basic method in Section I B for finding the Lagrangian configuration, we need to couple the eigenmodes to the of any circuit, when not including dissipation or mutual external degrees of freedom. Such external degrees of free- inductance, while the method in Section II B presents an dom could also be used to control or measure the system. alternative to the basic method. If mutual inductance While a non-transformed node flux can be controlled by is to be included, the method in Section II B must be coupling a single control line to the corresponding node used in order to obtain the Lagrangian. Having found the 43

Lagrangian one can change the node flux basis as in Sec- Draw circuit tion I B 3 or keep it in the original basis. The Hamiltonian and Label components can be found using Legendre transformation and then quantized as in Section I C 1. After asserting that the system is only weakly anharmonic it can be rewritten into If external fluxes: Label all nodes interacting harmonic oscillators perturbed by the anhar- Choose spanning and monicity, as in Section I D. Alternatively one can decide tree determine ground Draw on exact truncation as in Section I F 3. After changing to corresponding step operators the rotating wave approximation can be graph and applied if needed, as in Section I E. If the anharmonicity Add the energy label everything of each element is large enough the system can finally be truncated into qubits or qudits using either the methods in Section I F or to find Lagrangian Create F-matrices Section I F 3. If the dissipation is included in the analysis and find as in Section II C and the system is truncated to two- Find Hamiltonian Lagrangian levels, then the decoherence time can be found using the Change node by Legendre flux basis results in Section II C 6. transformation We note that the methods presented here are in no means exhaustive in regards to circuit analysis. Classical Quantize electrical circuit analysis has been performed for decades Hamiltonian by both and engineers, and much more infor- mation about this can be found in the existing literature. The methods presented here should therefore not be seen Assert system is as a limit to what can be done with superconducting in desired regime, circuits, but merely as a starting point for researchers e.g. transmon new to the field of superconducting electrical circuit anal- ysis. So where to go from here? If you want to explore

Recast to controlling and measuring superconducting circuits we interacting Decide on recommend Ref. [61] which discusses the coupling to mi- harmonic exact truncation crowave resonators. For more information on the method oscillators in Section I B and advanced method in Section II B see Refs. [50] and [58], respectively. Both also discuss how to analyse dissipation. For each of the examples, we Change into reference the original research, and finally for a recent step operators overview of the field see Ref. [11] which reviews recent Perform RWA state-of-the-art concepts. to eliminate fast oscillating terms Assert anharmonicity is sufficiently large

Truncate to Truncate to ACKNOWLEDGMENTS two-levels multi-levels (qubits) (qudits)

We wish to thank our numerous experimental and the- Find oretical collaborators and colleagues for discussions and decoherence collaborations on superconducting circuits over the past times, T1 and T2 few years, in particular D. Pestrosyan, M. Kjaergaard, W. Oliver, S. Gustavsson, and C. K. Andersen. Thank you Figure 24. Overview of the methods presented in this tutorial. also to all the students that have challenged us to provide Blue blocks indicates basic methods presented in Section I. clear and introductory explanations in this exciting field. Yellow blocks indicate optional steps, while red blocks indi- They are the real reason that we undertook the task of cates advanced methods presented in Section II. Round blocks writing this tutorial. This work is supported by the Dan- is assertions that must be satisfied before advancing in the ish Council for Independent Research and the Carlsberg flowchart. Foundation. 44

[1] Richard P. Feynman. Simulating physics with computers. Zhang, Hao Li, Yuxuan Li, Xiao Jiang, Lin Gan, Guang- International Journal of Theoretical Physics, 21(6):467– wen Yang, Lixing You, Zhen Wang, Li Li, Nai-Le Liu, 488, Jun 1982. Chao-Yang Lu, and Jian-Wei Pan. Quantum computa- [2] Seth Lloyd. Universal quantum simulators. Science, tional advantage using photons. Science, 370(6523):1460– 273(5278):1073–1078, 1996. 1463, 2020. [3] David P. DiVincenzo. The physical implementation of [11] P. Krantz, M. Kjaergaard, F. Yan, T. P. Orlando, S. Gus- quantum computation. Fortschritte der Physik, 48:771– tavsson, and W. D. Oliver. A quantum engineer’s guide 783, 2000. to superconducting qubits. Applied Physics Reviews, [4] Jonathan P. Dowling and Gerard J. Milburn. Quantum 6(2):021318, 2019. technology: the second quantum revolution. Philosophi- [12] M. A. Nielsen and I. L. Chuang. Quantum Computation cal Transactions of the Royal Society of London. Series and Quantum Information. Cambridge University Press, A: Mathematical, Physical and Engineering Sciences, Cambridge, UK, 2010. 361(1809):1655–1674, 2003. [13] Anargyros Papageorgiou and Joseph F. Traub. Measures [5] T. D. Ladd, F. Jelezko, R. Laflamme, Y. Nakamura, of quantum computing speedup. Phys. Rev. A, 88:022316, C. Monroe, and J. L. O’Brien. Quantum computers. Aug 2013. Nature, 464(7285):45–53, Mar 2010. [14] John Preskill. Quantum Computing in the NISQ era [6] Hannes Bernien, Sylvain Schwartz, Alexander Keesling, and beyond. Quantum, 2:79, August 2018. Harry Levine, Ahmed Omran, Hannes Pichler, Soon- [15] J. I. Cirac and P. Zoller. Quantum computations with won Choi, Alexander S. Zibrov, Manuel Endres, Markus cold trapped ions. Phys. Rev. Lett., 74:4091–4094, May Greiner, Vladan Vuleti´c,and Mikhail D. Lukin. Probing 1995. many-body dynamics on a 51-atom . [16] D. Leibfried, R. Blatt, C. Monroe, and D. Wineland. Nature, 551(7682):579–584, Nov 2017. Quantum dynamics of single trapped ions. Rev. Mod. [7] Aram W. Harrow and Ashley Montanaro. Quantum Phys., 75:281–324, Mar 2003. computational supremacy. Nature, 549(7671):203–209, [17] D. Porras and J. I. Cirac. Effective quantum spin systems Sep 2017. with trapped ions. Phys. Rev. Lett., 92:207901, May [8] Frank Arute, Kunal Arya, Ryan Babbush, Dave Bacon, 2004. Joseph C. Bardin, Rami Barends, Rupak Biswas, Sergio [18] and David Wineland. Entangled states of Boixo, Fernando G. S. L. Brandao, David A. Buell, Brian trapped atomic ions. Nature, 453(7198):1008–1015, Jun Burkett, Yu Chen, Zijun Chen, Ben Chiaro, Roberto 2008. Collins, William Courtney, Andrew Dunsworth, Ed- [19] H. H¨affner,C.F. Roos, and R. Blatt. Quantum com- ward Farhi, Brooks Foxen, Austin Fowler, Craig Gidney, puting with trapped ions. Physics Reports, 469(4):155 – Marissa Giustina, Rob Graff, Keith Guerin, Steve Habeg- 203, 2008. ger, Matthew P. Harrigan, Michael J. Hartmann, Alan [20] R. Blatt and C. F. Roos. Quantum simulations with Ho, Markus Hoffmann, Trent Huang, Travis S. Humble, trapped ions. Nature Physics, 8(4):277–284, Apr 2012. Sergei V. Isakov, Evan Jeffrey, Zhang Jiang, Dvir Kafri, [21] D. Jaksch and P. Zoller. The cold atom hubbard toolbox. Kostyantyn Kechedzhi, Julian Kelly, Paul V. Klimov, Annals of Physics, 315(1):52 – 79, 2005. Special Issue. Sergey Knysh, Alexander Korotkov, Fedor Kostritsa, [22] , Anna Sanpera, Veronica Ahufinger, David Landhuis, Mike Lindmark, Erik Lucero, Dmitry Bogdan Damski, Aditi Sen(De), and Ujjwal Sen. Ul- Lyakh, Salvatore Mandr`a,Jarrod R. McClean, Matthew tracold atomic gases in optical lattices: mimicking con- McEwen, Anthony Megrant, Xiao Mi, Kristel Michielsen, densed matter physics and beyond. Advances in Physics, Masoud Mohseni, Josh Mutus, Ofer Naaman, Matthew 56(2):243–379, 2007. Neeley, Charles Neill, Murphy Yuezhen Niu, Eric Os- [23] Immanuel Bloch, Jean Dalibard, and Wilhelm Zwerger. tby, Andre Petukhov, John C. Platt, Chris Quintana, Many-body physics with ultracold gases. Rev. Mod. Eleanor G. Rieffel, Pedram Roushan, Nicholas C. Ru- Phys., 80:885–964, Jul 2008. bin, Daniel Sank, Kevin J. Satzinger, Vadim Smelyan- [24] Christian Gross and Immanuel Bloch. Quantum simula- skiy, Kevin J. Sung, Matthew D. Trevithick, Amit tions with ultracold atoms in optical lattices. Science, Vainsencher, Benjamin Villalonga, Theodore White, 357(6355):995–1001, 2017. Z. Jamie Yao, Ping Yeh, Adam Zalcman, Hartmut [25] Florian Sch¨afer,Takeshi Fukuhara, Seiji Sugawa, Yosuke Neven, and John M. Martinis. Quantum supremacy Takasu, and Yoshiro Takahashi. Tools for quantum sim- using a programmable superconducting processor. Na- ulation with ultracold atoms in optical lattices. Nature ture, 574(7779):505–510, Oct 2019. Reviews Physics, 2(8):411–425, Aug 2020. [9] Hui Wang, Jian Qin, Xing Ding, Ming-Cheng Chen, [26] Daniel Loss and David P. DiVincenzo. Quantum com- Si Chen, Xiang You, Yu-Ming He, Xiao Jiang, L. You, putation with quantum dots. Phys. Rev. A, 57:120–126, Z. Wang, C. Schneider, Jelmer J. Renema, Sven H¨ofling, Jan 1998. Chao-Yang Lu, and Jian-Wei Pan. Boson sampling with [27] B. E. Kane. A silicon-based nuclear spin quantum com- 20 input photons and a 60-mode interferometer in a 1014- puter. Nature, 393(6681):133–137, May 1998. dimensional hilbert space. Phys. Rev. Lett., 123:250503, [28] Rogerio de Sousa, J. D. Delgado, and S. Das Sarma. Dec 2019. Silicon quantum computation based on magnetic dipolar [10] Han-Sen Zhong, Hui Wang, Yu-Hao Deng, Ming-Cheng coupling. Phys. Rev. A, 70:052304, Nov 2004. Chen, Li-Chao Peng, Yi-Han Luo, Jian Qin, Dian Wu, [29] Rutger Vrijen, Eli Yablonovitch, Kang Wang, Hong Wen Xing Ding, Yi Hu, Peng Hu, Xiao-Yan Yang, Wei-Jun Jiang, Alex Balandin, Vwani Roychowdhury, Tal Mor, 45

and David DiVincenzo. Electron-spin-resonance tran- qubit revisted to enhance cohrence and reproducibility. sistors for quantum computing in silicon-germanium Nature Communications, 7(12964), 8 2016. heterostructures. Phys. Rev. A, 62:012306, Jun 2000. [44] R. Barends, J. Kelly, A. Megrant, D. Sank, E. Jef- [30] L. C. L. Hollenberg, A. D. Greentree, A. G. Fowler, and frey, Y. Chen, Y. Yin, B. Chiaro, J. Mutus, C. Neill, C. J. Wellard. Two-dimensional architectures for donor- P. O’Malley, P. Roushan, J. Wenner, T. C. White, A. N. based quantum computing. Phys. Rev. B, 74:045311, Jul Cleland, and John M. Martinis. Coherent josephson 2006. qubit suitable for scalable quantum integrated circuits. [31] Andrea Morello, Jarryd J. Pla, Floris A. Zwanenburg, Phys. Rev. Lett., 111:080502, 8 2013. Kok W. Chan, Kuan Y. Tan, Hans Huebl, Mikko [45] William D. Oliver and Paul B. Welander. Materi- M¨ott¨onen, Christopher D. Nugroho, Changyi Yang, Jes- als in superconducting quantum bits. MRS Bulletin, sica A. van Donkelaar, Andrew D. C. Alves, David N. 38(10):816–825, 2013. Jamieson, Christopher C. Escott, Lloyd C. L. Hollen- [46] A V Shcherbakova, K G Fedorov, K V Shulga, V V berg, Robert G. Clark, and Andrew S. Dzurak. Single- Ryazanov, V V Bolginov, V A Oboznov, S V Egorov, V O shot readout of an electron spin in silicon. Nature, Shkolnikov, M J Wolf, D Beckmann, and A V Ustinov. 467(7316):687–691, Oct 2010. Fabrication and measurements of hybrid nb/al josephson [32] A. Imamoglu, D. D. Awschalom, G. Burkard, D. P. Di- junctions and flux qubits withπ-shifters. Superconductor Vincenzo, D. Loss, M. Sherwin, and A. Small. Quantum Science and Technology, 28(2):025009, jan 2015. information processing using quantum dot spins and [47] I. Tsioutsios, K. Serniak, S. Diamond, V. V. Sivak, cavity qed. Phys. Rev. Lett., 83:4204–4207, Nov 1999. Z. Wang, S. Shankar, L. Frunzio, R. J. Schoelkopf, and [33] Dirk Englund, David Fattal, Edo Waks, Glenn Solomon, M. H. Devoret. Free-standing silicon shadow masks for Bingyang Zhang, Toshihiro Nakaoka, Yasuhiko Arakawa, transmon qubit fabrication. AIP Advances, 10(6):065120, Yoshihisa Yamamoto, and Jelena Vuˇckovi´c.Controlling 2020. the spontaneous emission rate of single quantum dots [48] A.O Caldeira and A.J Leggett. Quantum tunnelling in a in a two-dimensional photonic crystal. Phys. Rev. Lett., dissipative system. Annals of Physics, 149(2):374 – 456, 95:013904, Jul 2005. 1983. [34] J. R. Petta, A. C. Johnson, J. M. Taylor, E. A. Laird, [49] Anthony J. Leggett. Quantum mechanics at the macro- A. Yacoby, M. D. Lukin, C. M. Marcus, M. P. Hanson, scopic level, pages 187–248. World Scientific, 1989. and A. C. Gossard. Coherent manipulation of coupled [50] Uri Vool and Michel Devoret. Introduction to quantum electron spins in semiconductor quantum dots. Science, electromagnetic circuits. International Journal of Circuit 309(5744):2180–2184, 2005. Theory and Applications, 45(7):897–934, 2017. [35] R. Hanson, L. P. Kouwenhoven, J. R. Petta, S. Tarucha, [51] J. Q. You and F. Nori. Superconducting circuits and and L. M. K. Vandersypen. Spins in few-electron quan- quantum information. Physics Today, 58(11), 2005. tum dots. Rev. Mod. Phys., 79:1217–1265, Oct 2007. [52] Iulia Buluta and Franco Nori. Quantum simulators. [36] Floris A. Zwanenburg, Andrew S. Dzurak, Andrea Science, 326(5949):108–111, 2009. Morello, Michelle Y. Simmons, Lloyd C. L. Hollenberg, [53] Iulia Buluta, Sahel Ashhab, and Franco Nori. Natural Gerhard Klimeck, Sven Rogge, Susan N. Coppersmith, and artificial atoms for quantum computation. Reports and Mark A. Eriksson. Silicon quantum electronics. Rev. on Progress in Physics, 74(10):104401, sep 2011. Mod. Phys., 85:961–1019, Jul 2013. [54] J. Q. You and Franco Nori. Atomic physics and quan- [37] M. V. Gurudev Dutt, L. Childress, L. Jiang, E. Togan, tum optics using superconducting circuits. Nature, J. Maze, F. Jelezko, A. S. Zibrov, P. R. Hemmer, and 474(7353):589–597, Jun 2011. M. D. Lukin. Quantum register based on individual [55] I. M. Georgescu, S. Ashhab, and Franco Nori. Quantum electronic and nuclear spin qubits in diamond. Science, simulation. Rev. Mod. Phys., 86:153–185, Mar 2014. 316(5829):1312–1316, 2007. [56] M. H. Devoret. Quantum fluctuations in electrical cir- [38] R. Hanson, O. Gywat, and D. D. Awschalom. Room- cuits. In Les Houches Session LXIII. Oxford University temperature manipulation and decoherence of a single Press, 1997. spin in diamond. Phys. Rev. B, 74:161203, Oct 2006. [57] M. H. Devoret, A. Wallraff, and J. M. Martinis. Super- [39] E. Knill, R. Laflamme, and G. J. Milburn. A scheme conducting qubits: A short review, 2004. for efficient quantum computation with linear optics. [58] Guido Burkard, Roger H. Koch, and David P. DiVin- Nature, 409(6816):46–52, Jan 2001. cenzo. Multilevel quantum description of decoherence in [40] T. B. Pittman, B. C. Jacobs, and J. D. Franson. Proba- superconducting qubits. Phys. Rev. B, 69:064503, Feb bilistic quantum logic operations using polarizing beam 2004. splitters. Phys. Rev. A, 64:062311, Nov 2001. [59] R. J. Schoelkopf and S. M. Girvin. Wiring up quantum [41] J. D. Franson, M. M. Donegan, M. J. Fitch, B. C. Jacobs, systems. Nature, 451(7179):664–669, Feb 2008. and T. B. Pittman. High-fidelity quantum logic oper- [60] John Clarke and Frank K. Wilhelm. Superconducting ations using linear optical elements. Phys. Rev. Lett., quantum bits. Nature, 453(7198):1031–1042, Jun 2008. 89:137901, Sep 2002. [61] S. M. Girvin. Circuit qed: Superconducting qubits cou- [42] T. B. Pittman, M. J. Fitch, B. C Jacobs, and J. D. pled to microwave photons. Quantum Machines: Mea- Franson. Experimental controlled-not logic gate for sin- surement and Control of Engineered Quantum Systems gle photons in the coincidence basis. Phys. Rev. A, – Lecture Notes of the Les Houches Summer School, 96, 68:032316, Sep 2003. 2014. [43] F. Yan, S. Gustavsson, A. Kamal, J. Birenbaum, A. P. [62] Jay M. Gambetta, Jerry M. Chow, and Matthias Steffen. Sears, D. Hover, T. J. Gudmundsen, D. Rosenberg, Building logical qubits in a superconducting quantum G. Samach, S. Weber, J. L. Yoder, T. P. Orlando, computing system. npj Quantum Information, 3(1):2, J. Clarke, A. J. Kerman, and W. D. Oliver. The flux Jan 2017. 46

[63] G Wendin. Quantum information processing with super- tive coupling of superconducting qubits in the strongly conducting circuits: a review. Reports on Progress in nonlinear regime. Phys. Rev. A, 95(5):52333, 2017. Physics, 80(10):106001, sep 2017. [83] M. Sch¨ondorfand F.K. Wilhelm. Nonpairwise inter- [64] Xiu Gu, Anton Frisk Kockum, Adam Miranowicz, actions induced by virtual transitions in four coupled Yu xi Liu, and Franco Nori. Microwave photonics with artificial atoms. Phys. Rev. Applied, 12:064026, Dec superconducting quantum circuits. Physics Reports, 718- 2019. 719:1 – 102, 2017. Microwave photonics with supercon- [84] Wolfgang Lechner, Philipp Hauke, and Peter Zoller. A ducting quantum circuits. quantum annealing architecture with all-to-all connec- [65] B. Peikari. Fundamentals of Network Analysis and Syn- tivity from local interactions. Science Advances, 1(9), thesis. Prentice-Hall, Englewood Cliffs, NJ, 1974. 2015. [66] A. O. Caldeira and A. J. Leggett. Influence of dissipation [85] Martin Leib, Peter Zoller, and Wolfgang Lechner. A on quantum tunneling in macroscopic systems. Phys. transmon quantum annealer: decomposing many-body Rev. Lett., 46:211–214, Jan 1981. Ising constraints into pair interactions. Quantum Science [67] D. J. Griffiths. Introduction to Electrodynamics. Pearson, and Technology, 1(1):15008, dec 2016. 1981. [86] J D Biamonte. Nonperturbative k-body to two-body com- [68] J. C. Gallop. SQUIDs, the Josephson Effect, and Super- muting conversion Hamiltonians and embedding problem conducting Electronics. Taylor and Francis Group, New instances into Ising spins. Phys. Rev. A, 77(5):52331, York, 1991. 2008. [69] L. N. Cooper. Bound electron pairs in a degenerate fermi [87] Alejandro Perdomo-Ortiz, Colin Truncik, Ivan Tubert- gas. Phys. Rev., 104:1189–1190, 11 1956. Brohman, Geordie Rose, and Al´anAspuru-Guzik. Con- [70] B.D. Josephson. Possible new effects in superconductive struction of model Hamiltonians for adiabatic quantum tunnelling. Physics Letters, 1(7):251 – 253, 1962. computation and its application to finding low-energy [71] B. D. Josephson. The discovery of tunnelling supercur- conformations of lattice protein models. Phys. Rev. A, rents. Proceedings of the IEEE, 62(6):838–841, 1974. 78(1):12320, jul 2008. [72] A. Barone and G. Paterno. Physics and Applications of [88] Alejandro Perdomo-Ortiz, Neil Dickson, Marshall Drew- the Josephson Effect. A Wiley-interscience publication. Brook, Geordie Rose, and Al´anAspuru-Guzik. Finding Wiley, 1982. low-energy conformations of lattice protein models by [73] Michel H. Devoret, John M. Martinis, and John Clarke. quantum annealing. Scientific Reports, 2:571, aug 2012. Measurements of macroscopic quantum tunneling out [89] Paul Dirac. The Priciples of Quantum Mechanics. Ox- of the zero-voltage state of a current-biased josephson ford University Press, UK, 4th edition, 1967. junction. Phys. Rev. Lett., 55:1908–1911, Oct 1985. [90] Jens Koch, Terri M. Yu, Jay Gambetta, A. A. Houck, [74] John R. Taylor. Classical Mechanics. University Science D. I. Schuster, J. Majer, Alexandre Blais, M. H. Devoret, Books, U.S.A., 2005. S. M. Girvin, and R. J. Schoelkopf. Charge-insensitive [75] Simon Panyella Pedersen, K. S. Christensen, and N. T. qubit design derived from the cooper pair box. Phys. Zinner. Native three-body interaction in superconducting Rev. A, 76:042319, Oct 2007. circuits. Physical Review Research, 1(3):033123, nov [91] J. J. Sakurai and J. J. Napolitano. Modern Quantum 2019. Mechanics. Pearson, USA, 2nd edition, 2011. [76] D. Marcos, P. Rabl, Enrique Rico, and Peter Zoller. Su- [92] L. Allen and J. H. Eberly. Optical Resonance and perconducting Circuits for Quantum Simulation of Dy- TwoLevel Atoms. Dover Publications, New York, 1987. namical Gauge Fields. Physical Review Letters, 111(11), [93] B. H. Bransden and C. J. Joachain. Physics of Atoms sep 2013. and Molecules. Prentice Hall, Essex, 2nd edition, 2003. [77] D. Marcos, Philippe Widmer, Enrique Rico, M. Hafezi, [94] Omar Gamel and Daniel F. V. James. Time-averaged P. Rabl, U.-J. Wiese, and Peter Zoller. Two-dimensional quantum dynamics and the validity of the effective hamil- lattice gauge theories with superconducting quantum tonian model. Phys. Rev. A, 82:052106, Nov 2010. circuits. Annals of Physics, 351:634–654, 2014. [95] Benjamin P. Lanyon, Marco Barbieri, Marcelo P. [78] Philipp Hauke, D. Marcos, Marcello Dalmonte, and Peter Almeida, Thomas Jennewein, Timothy C. Ralph, Zoller. Quantum Simulation of a Lattice Schwinger Kevin J. Resch, Geoff J. Pryde, Jeremy L. O’Brien, Model in a Chain of Trapped Ions. Physical Review X, Alexei Gilchrist, and Andrew G. White. Simplifying 3(4):041018, nov 2013. quantum logic using higher-dimensional hilbert spaces. [79] Seth Lloyd and Barbara M Terhal. Adiabatic and Hamil- Nature Physics, 5(2):134–140, Feb 2009. tonian computing on a 2D lattice with simple two-qubit [96] A. Fedorov, L. Steffen, M. Baur, M. P. da Silva, and interactions. New Journal of Physics, 18(2):23042, feb A. Wallraff. Implementation of a toffoli gate with su- 2016. perconducting circuits. Nature, 481(7380):170–172, Jan [80] A. Mezzacapo, Enrique Rico, C. Sab´ın,I. L. Egusquiza, 2012. L. Lamata, and Enrique Solano. Non-Abelian SU(2) [97] T. Bækkegaard, L. B. Kristensen, N. J. S. Loft, C. K. Lattice Gauge Theories in Superconducting Circuits. Andersen, D. Petrosyan, and N. T. Zinner. Realization Phys. Rev. Lett., 115(24):240502, dec 2015. of efficient quantum gates with a superconducting qubit- [81] N Chancellor, S Zohren, and P A Warburton. Circuit qutrit circuit. Scientific Reports, 9(1):13389, Sep 2019. design for multi-body interactions in superconducting [98] Niels Jakob Søe Loft, Morten Kjaergaard, Lasse Bjørn quantum annealing systems with applications to a scal- Kristensen, Christian Kraglund Andersen, Thorvald W. able architecture. npj Quantum Information, 3(1):21, Larsen, Simon Gustavsson, William D. Oliver, and Niko- 2017. laj T. Zinner. Quantum interference device for controlled [82] Dvir Kafri, Chris Quintana, Yu Chen, Alireza Shabani, two-qubit operations. npj Quantum Information, 6(1):47, John M Martinis, and Hartmut Neven. Tunable induc- May 2020. 47

[99] S. E. Rasmussen, K. S. Christensen, and N. T. Zinner. Mathematical Foundations. Springer, Germany, 1980. Controllable two-qubit swapping gate using supercon- [117] J. A. Schreier, A. A. Houck, Jens Koch, D. I. Schuster, ducting circuits. Phys. Rev. B, 99:134508, Apr 2019. B. R. Johnson, J. M. Chow, J. M. Gambetta, J. Majer, [100] Christopher Gerry and Peter Knight. Introductory Quan- L. Frunzio, M. H. Devoret, S. M. Girvin, and R. J. tum Optics. Cambridge University Press, Cambridge, Schoelkopf. Suppressing charge noise decoherence in England, 2005. superconducting charge qubits. Phys. Rev. B, 77:180502, [101] Erik Lucero, M. Hofheinz, M. Ansmann, Radoslaw C. May 2008. Bialczak, N. Katz, Matthew Neeley, A. D. O’Connell, [118] Ivan V. Pechenezhskiy, Raymond A. Mencia, Long B. H. Wang, A. N. Cleland, and John M. Martinis. High- Nguyen, Yen-Hsiang Lin, and Vladimir E. Manucharyan. fidelity gates in a single josephson qubit. Phys. Rev. The superconducting quasicharge qubit. Nature, Lett., 100:247001, Jun 2008. 585(7825):368–371, Sep 2020. [102] J. M. Chow, J. M. Gambetta, L. Tornberg, Jens Koch, [119] R. C. Jaklevic, John Lambe, A. H. Silver, and J. E. Lev S. Bishop, A. A. Houck, B. R. Johnson, L. Frun- Mercereau. Quantum interference effects in josephson zio, S. M. Girvin, and R. J. Schoelkopf. Randomized tunneling. Phys. Rev. Lett., 12:159–160, Feb 1964. benchmarking and process tomography for gate errors [120] T. P. Orlando, J. E. Mooij, Lin Tian, Caspar H. van der in a solid-state qubit. Phys. Rev. Lett., 102:090502, Mar Wal, L. S. Levitov, Seth Lloyd, and J. J. Mazo. Su- 2009. perconducting persistent-current qubit. Phys. Rev. B, [103] F. Motzoi, J. M. Gambetta, P. Rebentrost, and F. K. 60:15398–15413, 12 1999. Wilhelm. Simple pulses for elimination of leakage in [121] J. E. Mooij, T. P. Orlando, L. Levitov, Lin Tian, weakly nonlinear qubits. Phys. Rev. Lett., 103:110501, Caspar H. van der Wal, and Seth Lloyd. Josephson Sep 2009. persistent-current qubit. Science, 285(5430):1036–1039, [104] S. E. Rasmussen, K. Groenland, R. Gerritsma, 1999. K. Schoutens, and N. T. Zinner. Single-step implemen- [122] J. Q. You, X. Hu, S. Ashhab, and F. Nori. Low- tation of high-fidelity n-bit toffoli gates. Phys. Rev. A, decoherence flux qubit. Phys. Rev. B, 75:140515, 4 101:022308, Feb 2020. 2007. [105] David C. McKay, Christopher J. Wood, Sarah Sheldon, [123] Vladimir E. Manucharyan, Jens Koch, Leonid I. Glaz- Jerry M. Chow, and Jay M. Gambetta. Efficient z gates man, and Michel H. Devoret. Fluxonium: Single cooper- for quantum computing. Phys. Rev. A, 96:022330, Aug pair circuit free of charge offsets. Science, 326(5949):113– 2017. 116, 2009. [106] J. M. Gambetta, F. Motzoi, S. T. Merkel, and F. K. [124] Ioan M. Pop, Kurtis Geerlings, Gianluigi Catelani, Wilhelm. Analytic control methods for high-fidelity Robert J. Schoelkopf, Leonid I. Glazman, and Michel H. unitary operations in a weakly nonlinear oscillator. Phys. Devoret. Coherent suppression of electromagnetic dis- Rev. A, 83:012308, Jan 2011. sipation due to superconducting quasiparticles. Nature, [107] M. Kounalakis, C. Dickel, A. Bruno, N. K. Langford, and 508(7496):369–372, Apr 2014. G. A. Steele. Tuneable hopping and nonlinear cross-Kerr [125] Long B. Nguyen, Yen-Hsiang Lin, Aaron Somoroff, interactions in a high-coherence superconducting circuit. Raymond Mencia, Nicholas Grabon, and Vladimir E. npj Quantum Information, 4(1):38, feb 2018. Manucharyan. High-coherence fluxonium qubit. Phys. [108] J. A. Bondy and U. S. R. Monty. Graph Theory with Rev. X, 9:041041, Nov 2019. Applications. Elsevier Science Publishing Co., 1976. [126] Jonathan R. Friedman and D. V. Averin. Aharonov- [109] R. Penrose. A generalized inverse for matrices. Mathe- casher-effect suppression of macroscopic tunneling of matical Proceedings of the Cambridge Philosophical So- magnetic flux. Phys. Rev. Lett., 88:050403, Jan 2002. ciety, 51(3):406–413, 1955. [127] Benoit Dou¸cot and Julien Vidal. Pairing of cooper pairs [110] H.-P. Breuer and F. Peruccione. The theory of open in a fully frustrated josephson-junction chain. Phys. Rev. quantum systems. Oxford University Press, New York, Lett., 88:227005, May 2002. NY, US, 2002. [128] L. B. Ioffe and M. V. Feigel’man. Possible realization of [111] A.G. Redfield. The theory of relaxation processes. In an ideal quantum computer in josephson junction array. John S. Waugh, editor, Advances in Magnetic Reso- Phys. Rev. B, 66:224503, Dec 2002. nance, volume 1 of Advances in Magnetic and Optical [129] Alexei Kitaev. Protected qubit based on a superconduct- Resonance, pages 1 – 32. Academic Press, 1965. ing current mirror. arXiv:cond-mat/0609441, 2006. [112] Yuriy Makhlin, Gerd Sch¨on,and Alexander Shnirman. [130] Peter Brooks, Alexei Kitaev, and John Preskill. Pro- Quantum-state engineering with josephson-junction de- tected gates for superconducting qubits. Phys. Rev. A, vices. Rev. Mod. Phys., 73:357–400, May 2001. 87:052306, May 2013. [113] Yu. V. Nazarov and D. A. Bagrets. Circuit theory for [131] Peter Groszkowski, A Di Paolo, A L Grimsmo, A Blais, full counting statistics in multiterminal circuits. Phys. D I Schuster, A A Houck, and Jens Koch. Coherence Rev. Lett., 88:196801, Apr 2002. properties of the 0-πqubit. New Journal of Physics, [114] John M. Martinis, Michel H. Devoret, and John Clarke. 20(4):043053, apr 2018. Energy-level quantization in the zero-voltage state of [132] Andras Gyenis, Pranav S. Mundada, Agustin Di Paolo, a current-biased josephson junction. Phys. Rev. Lett., Thomas M. Hazard, Xinyuan You, David I. Schuster, 55:1543–1546, Oct 1985. Jens Koch, Alexandre Blais, and Andrew A. Houck. Ex- [115] V. Bouchiat, D. Vion, P. Joyez, D. Esteve, and M. H. perimental realization of an intrinsically error-protected Devoret. Quantum coherence with a single cooper pair. superconducting qubit, 2019. Physica Scripta, 1998(T76):165, 1998. [133] Joshua M. Dempster, Bo Fu, David G. Ferguson, D. I. [116] Josef Meixner, Friedrich W. Sch¨afke, and Gerhard Wolf. Schuster, and Jens Koch. Understanding degenerate Mathieu Functions and Spheroidal Functions and Their ground states of a protected quantum circuit in the 48

presence of disorder. Phys. Rev. B, 90:094518, Sep 2014. bus. Phys. Rev. Applied, 6:064007, Dec 2016. [134] K. S. Christensen, S. E. Rasmussen, D. Petrosyan, and [137] Fei Yan, Philip Krantz, Youngkyu Sung, Morten Kjaer- N. T. Zinner. Coherent router for quantum networks with gaard, Daniel L. Campbell, Terry P. Orlando, Simon superconducting qubits. Phys. Rev. Research, 2:013004, Gustavsson, and William D. Oliver. Tunable coupling Jan 2020. scheme for implementing high-fidelity two-qubit gates. [135] Yu Chen, C. Neill, P. Roushan, N. Leung, M. Fang, Phys. Rev. Applied, 10:054062, Nov 2018. R. Barends, J. Kelly, B. Campbell, Z. Chen, B. Chiaro, [138] S. E. Rasmussen and N. T. Zinner. Simple implementa- A. Dunsworth, E. Jeffrey, A. Megrant, J. Y. Mu- tion of high fidelity controlled-iswap gates and quantum tus, P. J. J. O’Malley, C. M. Quintana, D. Sank, circuit exponentiation of non-hermitian gates. Phys. Rev. A. Vainsencher, J. Wenner, T. C. White, Michael R. Research, 2:033097, Jul 2020. Geller, A. N. Cleland, and John M. Martinis. Qubit ar- [139] J. K. Pachos. Introduction to Topological Quantum Com- chitecture with high coherence and fast tunable coupling. putation. Cambridge University Press, New York, 1st Phys. Rev. Lett., 113:220502, Nov 2014. edition, 2012. [136] David C. McKay, Stefan Filipp, Antonio Mezzacapo, [140] P. Zanardi and M. Rasetti. Holonomic quantum compu- Easwar Magesan, Jerry M. Chow, and Jay M. Gambetta. tation. Physics Letters A, 264(2):94 – 99, 1999. Universal gate for fixed-frequency qubits via a tunable